Peds Case Files

अब Quizwiz के साथ अपने होमवर्क और परीक्षाओं को एस करें!

38.2 A 15-year-old adolescent girl has 2 days of nasal congestion and cough. Upon auscultation of her back, you find the lesions noted. Which of the following is the most likely etiology for her condition? (picture of child's back has multiple round bruises)

A. This adolescent has multiple perfectly circular lesions on her back consistent with cupping; when asked, she gives the history of cupping. Physical abuse injuries likely would not be identical in appearance. Patients with DIC will have significant systemic manifestation, and the pattern of ecchymoses would not be symmetrical. Coining causes ecchymosis in a linear pattern.

8.2 A term male is born via repeat cesarean section to a 30-year-old woman. Immediately after birth he has mild respiratory distress. Chest auscultation in the delivery room reveals clear breath sounds. Which of the following is the most appropriate next step? A Endotracheal intubation with direct suction. B Begin intravenous antibiotic therapy. C Deliver surfactant therapy. D Observe and administer supplemental oxygen as needed. E Bag-mask ventilation.

D. Because this infant most likely has TTN, the next step is to observe and administer supplemental oxygen, as needed.

51.2 A 12-year-old boy is suspected of having SLE because of his malar rash, migratory polyarticular arthritis, positive antinuclear antibody (ANA), and positive anti-ds DNA. His complete blood count (CBC) is normal. He is normotensive and a review of systems is positive only for arthritis and the rash. Which of the following tests is indicated in his evaluation? A Rapid plasma reagin (RPR) B Hepatitis panel C Skin biopsy D Urinalysis with microscopy E Rheumatoid factor (RF)

D. Identification of nephritis and proteinuria is essential at the time of diagnosis of SLE. Renal disease is a common cause of mortality for patients with SLE.

12.4 Which of the following is a potentially distinguishing feature of osteogenesis imperfecta versus rickets? A Poor muscle tone B Easily fractured long bones C Increased susceptibility to respiratory infections D Small stature E Blue sclera

E. Osteogenesis imperfecta is a congenital bone disorder that is caused by defective connective tissue formation secondary to collagen deficiency. Diagnosis is confirmed via collagen or DNA testing. Inheritance can be either autosomal dominant or autosomal recessive (depending on the type). There are eight different types of osteogenesis imperfecta with varying degrees of disability, but many of the features bare similarities to rickets. A few exceptions to this, however, are bluish sclera, triangular facies, and degrees of hearing loss.

1.4 A term female infant is born vaginally after an uncomplicated pregnancy. She appears normal but has respiratory distress when she stops crying. When crying she is pink; when not she makes vigorous respiratory efforts but becomes dusky. Which of the following is the likely explanation for her symptoms? A Choanal atresia B Diaphragmatic hernia C Meconium aspiration D Neonatal narcosis

A. Infants are obligate nose breathers until about 4 months of age. When crying they can breathe through their mouth, but they must have a patent nose when quiet. Choanal atresia is identified by passing a feeding tube through each nostril or by identification of clouding on cold metal held under the infant's nose. Should choanal atresia be diagnosed, endotracheal intubation bypasses the airway obstruction until surgical repair can be completed.

35.2 An expectant mother accompanies her son for a routine visit. She states that she was told that her unborn child may have posterior urethral valves. Which of the following statements is true about the condition? A It occurs with equal frequency in males and females. B The unborn child will need a CT scan immediately after birth. C No postnatal evaluation or follow-up is required if the neonate has a normal respiratory function at birth. D The risk of end-stage renal disease or chronic renal insufficiency is 25% to 30%. E Use of prophylactic antibiotic therapy may preclude the need for more invasive treatment.

D. The risk of end-stage renal disease or chronic renal insufficiency is 25% to 30%. Postnatal evaluation includes VCUG or postnatal USG.

18.1 An 18-month-old girl is seen in the clinic for cough and fever. Her weight is in the third percentile. The mother reports she is concerned her daughter is around "toxic mold" because she has had five to six prior episodes of bronchitis since they moved to a new apartment at 6 months of age. She states that albuterol and an antibiotic are always given for treatment, and symptoms resolve in 2 weeks. A chest radiograph is obtained. What finding on the radiograph would prompt you to perform a sweat chloride test? A An enlarged cardiac silhouette B Absent thymus C Bronchiectasis D Dextrocardia E Hilar lymphadenopathy

C. Bronchiectasis occurs as a sequela to impaired mucus clearance combined with inflammation and injury to the bronchial walls. CF is the most common noninfectious and chronic cause of this finding; sweat chloride testing is a standard for diagnosis. The other findings are not characteristics of CF.

17.3 A female is born via spontaneous vaginal delivery at 28 weeks of gestation because of preterm labor. Which of the following would be a risk factor for CP? A Hypoglycemia B Apnea of prematurity C Intraventricular hemorrhage D Blood transfusion E Mechanical ventilation

C. Intraventricular hemorrhage can later result in periventricular leukomalacia, which has been associated with an increased chance of CP.

2.3 Which of the following is NOT a symptom of hypoglycemia? A Tachypnea B Hypothermia C Poor feeding D Vomiting E Jitteriness

D. Vomiting typically is not a symptom of hypoglycemia.

4.4 A 7-day-old infant is seen in the emergency department for fever and poor feeding. The baby was delivered vaginally 2 hours after the mother arrived to hospital. The delivery was a 36 weeks of gestation and the birth weight was 2900 g. Maternal laboratory test results were negative. The most likely organism causing this patient's symptoms is: A Group B Streptococcus (GBS) B Listeria monocytogenes C Staphylococcus aureus D Streptococcus pneumoniae E Haemophilus influenza

A. GBS is the most common pathogen to cause neonatal sepsis in infants aged 0 to 3 months.

3.4 A 30-hour-old full-term infant has facial and chest jaundice. He is breast-feeding well and has an otherwise normal examination. His bilirubin level is 15.5 mg/dL. Which of the following is the most appropriate course of action? A Recommend cessation of breast-feeding for 48 hours and supplement with formula. B Start phototherapy. C Wait for 6 hours and retest the serum bilirubin level. D Start an exchange transfusion. E No action is needed.

B. Although the etiology of the hyperbilirubinemia must be investigated, phototherapy should be started.

22.1 Which of the following is NOT an important part of the initial evaluation of a heart murmur? A Chest radiograph (CXR) B Arterial blood gas (ABG) C Echocardiogram D Pulse oximetry E Electrocardiogram (ECG)

B. An arterial blood gas gives information about the patient's oxygenation status but does not help identify if a heart murmur is pathologic.

14.1A 6-week-old boy, born by vaginal delivery after an uncomplicated term gestation, has experienced cough and "fast breathing" for 2 days. His mother relates that he has a 1-week history of nasal congestion and watery eye discharge, but no fever or change in appetite. He has a temperature of 99.4°F (37.4°C) and a respiratory rate of 44 breaths/min. He has nasal congestion, clear rhinorrhea, erythematous conjunctivae bilaterally, and watery, right eye discharge. His lungs demonstrate scattered crackles without wheezes. Which of the following is the most likely pathogen? A Chlamydia trachomatis B Listeria monocytogenes C Respiratory syncytial virus D Rhinovirus E Streptococcus pneumoniae

A. Cough and increased respiratory effort in an afebrile infant with eye discharge are consistent with Chlamydia. Transmission typically occurs during vaginal delivery. Approximately 25% of infants born to mothers with Chlamydia develop conjunctivitis; about half of these develop pneumonia. Most infants present with respiratory infection in the second month of life, but symptoms can be seen as early as the second week. Inner eyelid swabs are sent for polymerase chain reaction (PCR), and oral erythromycin or sulfisoxazole (latter only in infants >2 months) is given for 2 weeks for either conjunctivitis or pneumonia.

36.1 In what position does the patient with nursemaid's elbow hold his arm? And how does the physician maneuver the child's arm for treatment of nursemaid's elbow? A Flexed and pronated, supinate B Flexed and supinated, pronate C Extended and pronated, supinate D Extended and supinated, pronate

A. A child with nursemaid's elbow holds the arm close to his body with the elbow flexed and forearm pronated. Treatment consists of supinating the child's forearm with the elbow in a flexed position while applying pressure over the radial head.

10.2 A 13-month-old child is noted to be at the 25th percentile for weight, the 10th percentile for height, and less than the 5th percentile for head circumference. She was born at term. She was noted to have a small head at birth, be developmentally delayed throughout her life, and have required cataract surgery shortly after birth. She currently takes phenobarbital for seizures. Caloric intake has been deemed appropriate by history, and neither frequent emesis nor excessive stooling is reported. Her examination is remarkable for a small head and liver enlargement on abdominal palpation. Which of the following would most likely explain this child's small size? A Congenital infection B Chromosomal abnormality C Metabolic disorder D Gastrointestinal dysmotility E Increased intracranial pressure

A. Developmental delay, intrauterine growth retardation (including microcephaly), cataracts, seizures, hepatosplenomegaly, prolonged neonatal jaundice, and purpura at birth are consistent with congenital cytomegalovirus (CMV) or toxoplasmosis infection. Calcified brain densities of CMV typically are found in a periventricular pattern; in toxoplasmosis, they are found scattered throughout the cortex.

8.1 A term male is born to a 33-year-old woman who had little prenatal care. Immediately after birth he has cyanosis and respiratory distress. Chest auscultation in the delivery room reveals right-sided heart sounds and absent left-sided breath sounds. Which of the following is the most appropriate next step? A Assess the abdomen to evaluate for possible congenital diaphragmatic hernia. B Order a computed tomography of the chest. C Order ultrasonography of the chest. D Perform a needle thoracostomy for possible pneumothorax. E Prepare the infant for extracorporeal membrane oxygenation (ECMO).

A. Evaluation of neonates born with respiratory distress and unilateral breath sounds includes an abdominal examination. With asymmetrical breath sounds, pneumothorax and congenital diaphragmatic hernia (CDH) are considered. This infant's scaphoid abdomen and the presence of bowel sounds in the chest suggest CDH; needle thoracostomy is avoided because intestinal perforation may occur. The patient is stabilized and the need for ECMO is ascertained after the infant's initial therapy response is evaluated. Many cases of CDH are diagnosed by prenatal ultrasound. Infants with CDH do not respond to typical steps of neonatal resuscitation, and often have worsening respiratory status upon bag-mask ventilation (BMV), ultimately requiring intubation. The diagnosis of CDH can be made by locating on plain imaging the nasogastric feeding tube in the chest where the stomach has been displaced from the abdomen.

35.1 A 3-month-old boy presents with fever without a source. As part of his evaluation a urinalysis is performed; a urinary tract infection (UTI) is suspected. Which of the following is the best next step? A If the urine culture reveals UTI, renal ultrasonography (USG) and voiding cystourethrogram (VCUG) should be performed. B VCUG should be performed only after a second UTI is diagnosed. C Antibiotics should be initiated after urine culture and sensitivities are obtained. D Renal biopsy should be performed. E Preferred methods of collection for urine culture for this infant include midstream clean-catch and bag urine.

A. For any male infant with a UTI, evaluation of anatomy and function is necessary. The preferred methods of urine collection include bladder catheterization and suprapubic bladder aspiration. Antimicrobial therapy is started empirically while awaiting urine culture and sensitivity results.

3.2 An 8-day-old infant continues to have jaundice which was first noted on the second day of life; his latest total and direct bilirubin levels are 12.5 and 0.9 mg/dL, respectively. The baby and the mother have type O positive blood, the direct and indirect Coombs tests are negative, the infant's reticulocyte count is 15%, and a smear of his blood reveals no abnormally shaped cells. He is bottle-feeding well, produces normal stools and urine, and has gained weight well. Which of the following remains in the differential diagnosis? A Gilbert syndrome B Disseminated intravascular coagulation (DIC) C Spherocytosis D Polycythemia E An undiagnosed blood group isoimmunization

A. Gilbert syndrome would present with a negative Coombs test, a normal (or low) hemoglobin, a normal (or slightly elevated) reticulocyte count, and prolonged hyperbilirubinemia. Red cell morphology would be abnormal in DIC and spherocytosis, polycythemia would present with an elevated hemoglobin level (that listed above is normal for a newborn), and blood group isoimmunization would present with a positive Coombs test.

6.1 A 10-day-old infant presents to clinic with a painful, red vesicular rash in the diaper area. He is mildly fussy but afebrile, and he has good oral intake. Which of the following is the most appropriate management of this infant? A Hospitalize the patient, obtain HSV surface and vesicle fluid cultures and CSF for HSV culture and PCR, and initiate intravenous acyclovir. B Order an EEG and brain MRI immediately. C Perform a Tzanck smear and send the patient home if it is negative. D Prescribe an antifungal cream and follow-up by telephone in 24 hours. E Schedule an appointment with a pediatric dermatologist.

A. In contrast to children and adults, neonates with suspected herpes skin lesions require parenteral antiviral therapy to prevent more serious sequelae as well as CSF analysis to define the extent of disease. HSV PCR of the CSF is the standard for diagnosis. Tzanck smears have low sensitivity and EEG may have nonspecific findings. The absence of fever does not indicate that the rash is from a benign etiology.

2.4 A term boy born to a mother with insulin-dependent pregestational diabetes has a bedside capillary glucose of 32 mg/dL at 1 hour of life. He is awake and has normal vital signs. Which of the following is the most appropriate next step in management? A Instruct the mother to breast-feed him and recheck the glucose in 30 minutes. B Place an IV and administer glucose. C Recheck the glucose in 1 hour. D Measure his serum insulin level. E Take him to the nursery for observation.

A. Infants of diabetic mothers are at risk for hypoglycemia and should feed within the first hour of birth and glucose should be measured 30 minutes later. In this infant's case, he has not fed yet, so he should breast-feed and have the glucose remeasured. IV glucose would be needed at this time only if he has symptoms of hypoglycemia. Rechecking the glucose in an hour without feeding is withholding treatment and not appropriate; the latter choices also do not include providing the appropriate treatment.

19.2 A father brings to the clinic his 6-year-old son who currently is undergoing induction chemotherapy for ALL. The school will not allow the child to register until his immunizations are up-to-date. Which of the following is the best course of action? A Call the school nurse or principal to inform him or her that this child should not receive immunizations while he is taking chemotherapy. B Update all immunizations except for measles, mumps, and rubella (MMR) and varicella. C Update all immunizations except for oral polio vaccine. D Update all immunizations. E Call the school nurse or principal to inform him or her that this child will never receive immunizations because of the alteration in his immune system.

A. Live virus vaccines are contraindicated for the child with ALL (and all members of the household) during chemotherapy and for at least 6 months after completion of treatment. Although the viruses in the vaccine are attenuated, immunosuppression from treatment can be profound and viral disease can result. Immunizations without live virus (diphtheria, tetanus, inactivated poliovirus vaccine, hepatitis A and B) are not absolutely contraindicated in this case, but the immunosuppression with chemotherapy often inhibits antibody responses.

13.4 Which of the following statements about routine procedures for a patient with SCD is accurate? A All children with SCD have baseline and then periodic complete blood count (CBC) and reticulocyte measurement screenings beginning at about 2 months of age. B To reduce the risk of sepsis, polysaccharide pneumococcal 23 vaccines are administered at 2, 4, and 6 months of age. C To identify new infiltrates, chest radiographs are obtained at all routine visits beginning at about 12 months of age. D Yearly gallbladder ultrasounds are indicated beginning in adolescence to identify the presence of stones. E Human papilloma virus vaccines are contraindicated in the SCD population.

A. Patients with SCD require baseline and periodic blood counts as described. The 23-valent polysaccharide pneumococcal vaccine is initiated at 2 years of age, whereas the 13-valent conjugate pneumococcal vaccine is administered at the younger ages outlined. Chest radiographs typically are obtained at approximately 2 years of age and periodically thereafter for screening purposes, for recent acute chest syndrome, or if the child has chronic cardiac or pulmonary disease. Ultrasounds of the gallbladder are reserved for patients with symptoms referable to that area.

18.3 A 10-year-old Caucasian boy has a history of recurrent sinusitis and multiple episodes of pneumonia. You suspect CF and order a sweat chloride test. The sweat electrolyte test result is within the normal range. What is your next step in management? A Perform DNA testing for CFTR (cystic fibrosis transmembrane conductance regulator) gene mutations. B Perform a pH probe test for gastroesophageal reflux. C Referral to pulmonologist. D Reassure parents that he does not have CF. E Place him on a high-calorie, high-protein diet.

A. The child has recurrent upper and lower respiratory tract infections suggesting CF. The sweat chloride test can yield falsely low values, so the next step would be to perform DNA testing to identify any of the common CFTR mutations. Negative sweat chloride test results do not exclude CF.

22.3 Which of the following statements is true about a ventricular septal defect (VSD)? A A murmur may not be present at birth. B Surgical repair is always required. C A continuous machinery-like murmur is characteristic. D A larger defect will have a louder murmur than a small defect. E A normal ECG excludes the presence of a VSD.

A. The murmur of a VSD is due to turbulent blood flow across the defect. When pulmonary vascular resistance is high enough, blood cannot flow from left to right across the defect. Once the pulmonary pressure drops to normal levels, which is typically by 2 to 6 months of age, the murmur becomes audible. The murmur is holosystolic and usually smaller defects create more turbulence which in turn, produces a louder murmur. Only large defects require surgical repair. The ECG is usually normal for small VSDs.

35.3 In which of the following neonates should the diagnosis of posterior urethral valves be considered? A A newborn with abdominal distension and respiratory distress with poor lung volumes on chest radiograph (CXR). B A newborn with abdominal distension bloody stools and air in the portal venous system on CXR. C A newborn with frequent episodes of apnea and bradycardia who is hypothermic. D A newborn with increased work of breathing with feedings, frequent desaturations, and perioral cyanosis. E A newborn who is large for gestational age and appears jittery on examination.

A. Posterior urethral valves can cause oligohydramnios, which can lead to lung hypoplasia. Some infants may even have urinary retention in the bladder or urinary ascites. Lung hypoplasia will cause respiratory distress and will be evident on chest radiograph. Abdominal distension, bloody stools, and portal venous air should raise suspicion for necrotizing enterocolitis. This typically occurs in premature infants after feedings have been initiated. Infant sepsis can have very subtle signs, such as frequent episodes of apnea and bradycardia as well as feeding intolerance. In infants fever should always prompt a workup for sepsis; however, hypothermia can be a sign of sepsis in a newborn as well. Infants who present with increased work of breathing with feeds and cyanosis should undergo evaluation for cyanotic heart disease. These infants can appear cyanotic when agitated or when stressed because of intracardiac shunting. Infants who appear jittery on examination should be evaluated for hypoglycemia. Infant who are small for gestational age, large for gestational age, or infants of diabetic mother are at increased risk for developing hypoglycemia within the first few hours of life.

30.4 The father of a 4-year-old girl calls your clinic to report her second febrile seizure. He states that this seizure was identical to the first one that happened 4 months ago: she developed an elevated temperature and within a short time had a generalized convulsion lasting 90 seconds. She was sleepy for approximately 2 minutes afterward. Upon awaking, she was given ibuprofen. She is now running around the house, chasing the family's chihuahua. The parents wonder if she needs to take anticonvulsants now that she has had another seizure. You should tell the father which of the following? A Febrile seizures frequently are recurrent but usually have no significant long-term effect. B You will prescribe an anticonvulsant because it will reduce the risk of future epilepsy. C You will order an EEG and CT scan of her head to be done on an outpatient basis. D He needs to take his daughter to the hospital for inpatient admission. E He should stop the ibuprofen and observe the fever curve.

A. Some children will develop recurrent febrile seizures. Anticonvulsants will decrease the risk of further febrile seizures, but they do not decrease the risk of developing epilepsy. The possible adverse reactions with antiepileptic medications are numerous, including severe allergic reactions and interference with school performance; often the benefit is not worth the risk. Fever reduction with medications is generally encouraged in children with a febrile seizure history. Hospital admission and diagnostic studies are not necessary in simple febrile seizures.

51.4 A 6-year-old boy is seen by the pediatrician for evaluation of fatigue and a 5-lb weight loss noted over the previous 4 months. He has complained of different sites of arthralgia over the same time interval and the mother has noticed the knee and then the hands appeared swollen on various occasions. After going outdoors, he has been developing pink papules on the sun-exposed areas of his body that he reports "sting." His urinalysis and CBC are normal. What are the most appropriate next diagnostic tests? A Antinuclear antibody (ANA), anti-dsDNA B Gram stain and culture of synovial fluid C Chest radiograph and electroencephalogram (EEG) D Rapid strep test and echocardiogram E Rheumatoid factor (RF) and hepatitis panel

A. The patient has two criteria for SLE: nonerosive arthritis and photosensitivity. If he is ANA and anti-DNA positive, he will have four of the criteria and should be referred to a rheumatologist.

37.1 A 2-year-old girl has a rash. She was well until 2 weeks prior when she had fever and upper respiratory tract infection (URI) symptoms that resolved without treatment. On examination, she has petechiae on her upper and lower extremities and trunk. Her platelet count is 25,000/mm3. Her white blood cell (WBC) count is 9000/mm3 and hemoglobin level is 11 mg/dL. Which of the following is the best next step in management? A Obtain a review of the peripheral blood smear. B Administer intravenous immunoglobulin. C Send a blood culture and begin empiric antimicrobial therapy. D Order a platelet transfusion. E Arrange for bone marrow biopsy.

A. This child has the classic immune thrombocytopenic purpura (ITP) features of isolated thrombocytopenia in a well-appearing child. An examination and peripheral blood smear are necessary. If no lymphadenopathy or organomegaly is found, the peripheral blood smear is normal, and there is no evidence of severe bleeding, initial management includes close observation and a protective environment.

44.1 An 8-year-old boy has short stature. He has no significant past medical history and his physical examination is normal. The growth curve shows growth parallel to a growth line at the fifth percentile. His father is 5 ft 4 in (163 cm), and his mother is 5 ft 1 in (155 cm). Radiograph of the left wrist shows a bone age equal to the chronological age. Which of the following is the most appropriate next step? A Reassure the parents that the child has normal prepubertal development. B Order thyroid function tests. C Order chromosomal analysis. D Refer to pediatric endocrinology for evaluation. E Obtain further radiographic imaging.

A. This child likely has familial short stature because both his parents are short. Bone age does not show catch-up potential in this case. Additional imaging or laboratory evaluations would not be necessary in this case because this child has no other symptoms aside from short stature. Reassurance is the most appropriate choice.

30.1 Paramedics bring a 7-month-old infant to the ED with seizure activity. The father reports the infant was in a normal state of health until approximately 3 days ago when she developed a febrile illness, diagnosed by her physician as a viral upper respiratory tract infection. Approximately 30 minutes ago she began having left arm jerking, which progressed to whole-body jerking. The episode spontaneously ceased on the way to the hospital. Vital signs include heart rate 90 beats/min, respiratory rate 25 breaths/min, and temperature 100.4°F (38°C). Your examination reveals a sleeping infant in no respiratory distress. The child's anterior fontanelle is full. The oropharynx is clear, and crusted mucus is found in the nares. The tympanic membranes are dark and without normal landmarks. The lungs are clear, and the heart and abdominal examinations are normal. She has a bruise over the occiput and several parallel bruises along the spine. Which of the following is the best next step in management? A Computed tomography (CT) of the head B Electroencephalogram (EEG) C Lumbar puncture D Observation E Phenobarbital

A. This child's history is worrisome for trauma. The fontanelle is full, bruises are found along the spine and on the occiput, and she has hemotympanum. A CT scan is of paramount importance; this child likely had a seizure from acute intracranial hemorrhage associated with physical abuse. Although this child is febrile and within the proper febrile seizure age range, the history and physical findings are more consistent with a diagnosis other than febrile seizure. Performing a lumbar puncture (LP) in a patient who may have increased intracranial pressure is not advisable, an EEG would probably not reveal the diagnosis, and phenobarbital is not immediately necessary in a patient who is not actively seizing.

21.2 A mother presents to the emergency room with her 6-month-old daughter late at night after she noticed her to be breathing fast for 1 minute, then seemed to stop breathing for 1 minute, and became limp, pale, and unresponsive. The mother attempted to give mouth-to-mouth breaths for a few seconds and her daughter then began to cry and her breathing and appearance normalized. Your next best step is to: A Perform a thorough history and physical examination, obtain basic laboratory tests, and admit to the hospital for workup of an ALTE. B Reassure the mother that her infant looks healthy and because the symptoms have resolved, discharge home. C Perform a complete blood count (CBC), chest x-ray (CXR), and discharge home if all are normal. D Instruct the mother to follow-up with the pediatrician to get an apnea monitor and pulse oximeter so that she will know if similar symptoms occur again and if they are actually life threatening. E Tell the mother this was a near SIDS event and instruct her on measures to prevent SIDS.

A. This clinical scenario is typical for an ALTE and is defined by the caregiver's history. It is not required to have other equipment or laboratory test results indicate an abnormality to meet the definition for the symptom. It requires evaluation to find the etiology.

34.3 A 6-week-old male infant has projectile emesis after feeding. He has an olive-shaped abdominal mass on abdominal examination. Which of the following statements is accurate? A He likely has hypochloremic metabolic alkalosis. B He likely has metabolic acidosis. C This condition is more common in female infants. D He should be restarted on feeds when the vomiting resolves. E He likely will develop diarrhea.

A. This infant has the features of pyloric stenosis, a condition four times more common in males and more common in first-born children. Affected infants usually present between the third and eighth weeks of life with increasing projectile emesis. Abdominal examination may reveal an olive-shaped mass and visible peristaltic waves. Serum electrolyte levels usually reveal hypochloremic metabolic alkalosis. Ultrasonography is useful in confirming the diagnosis.

47.4 An 11-year-old boy is seen in the physician's office complaining of right hip pain when he walks. On physical examination, he has an antalgic gait, pain on passive range of motion of his right hip, and external rotation during hip flexion. What of the following is the next best step in management? A Place the patient in non-weight-bearing status and obtain bilateral hip films. B Send a referral to orthopedic surgery. C Obtain synovial fluid by arthrocentesis of the right hip joint. D Recommend nonsteroidal anti-inflammatory drugs (NSAIDs) and follow-up in 2 weeks. E Obtain bilateral knee films.

A. This patient has characteristic clinical symptomatology of SCFE: a male in early adolescence who demonstrates a gait characterized by pain during weight bearing and a key diagnostic clue on physical examination of external rotation of the hip during hip flexion. Prior to obtaining radiographs in the patient with suspected SCFE, the patient is immediately made non-weight bearing to prevent further displacement.

47.2 A 2-year-old boy is seen by the pediatrician for a 1-week history of limping. His father says he slipped and fell after playing outside the previous week. On physical examination, his left thigh is swollen and he has an antalgic gait. You notice bruises on his buttocks and inner thighs. His father comments that he is very active and always gets bumps and bruises. The patient's past medical history is significant for two previous ER visits for fractures. Which of the following would be the best diagnostic test for this patient's condition? A Skeletal survey B Anteroposterior (AP) and frog-legged radiographs of the hip C Complete blood count (CBC), coagulation studies, von Willebrand factor activity, serum factor 8 and 9 level D Ultrasound of the hip joint E Observation and reassurance

A. This patient requires an immediate evaluation for nonaccidental trauma (NAT). After a thorough history and physical examination, the best diagnostic tool for NAT is a skeletal survey. Such patients typically are admitted to the hospital, and a skeletal survey will be done to evaluate for active or healing fractures. Child protective services should also be contacted. Important features that support NAT are delay in seeking medical care, inconsistent or vague history, and physical examination findings that appear as bruises in the shape of an object (hand, spatula), burns in the shape of the end of a cigarette, and bruising behind the ear, neck, trunk, or inner thighs.

16.4 A 1-month-old boy has a fever to 102.7°F (39.3°C), is irritable, has diarrhea, and has not been eating well. On examination he has an immobile red TM that has pus behind it. Which of the following is the most appropriate course of action? A Admission to the hospital with complete sepsis evaluation B Intramuscular ceftriaxone and close outpatient follow-up C Oral amoxicillin-clavulanate D Oral cefuroxime E High-dose oral amoxicillin

A. Very young children with OM (especially if irritable or lethargic) are at higher risk for bacteremia or other serious infection. Hospitalization and parenteral antibiotics often are needed.

14.4 A 12-year-old boy presents to your clinic complaining of 1 week of fever, cough, and chest pain. He recently went on a boy scouts trip to Arizona, where he camped outside for 2 weeks, and excitedly describes how he became trapped in a "haboob" (massive dust storm) for an hour with his troop mates. His mother mentions that one of the other boys in his troop developed an unidentified, painful rash on his shins shortly after returning from the trip. The patient has a temperature of 101°F (38.3°C), but otherwise stable vital signs, and his physical examination reveals bibasilar rales. Which of the following is the most likely pathogen? A Mycobaterium tuberculae B Coccidioides immitis C Myocoplasma pneumoniae D Staphylococcus aureus E Influenza

B. Coccidioides spores often are found in the soil in the southwestern Unites States. Coccidioidomycosis typically results from inhalation of spores during dust-generating events. Symptoms may include constitutional (fatigue, weight loss), isolated cutaneous (erythema nodosum), and LRTI, akin to community-acquired pneumonia (cough, pleuritic chest pain). The incubation period ranges from 1 to 4 weeks. Treatment for symptomatic infection is with systemic antifungal (fluconazole, itraconazole).

47.3 A new 16-year-old patient is seen by the pediatrician before beginning her school year; her family has just moved into town. Her past medical history is significant recent hospitalization for slipped capital femoral epiphysis (SCFE) 6 months prior where she underwent internal pinning of her left hip. Her growth curve shows declining growth velocity from 50th percentile several years ago to the 15th percentile currently. Her weight is in the 85th percentile. Her mother notes that she often is tired and sleeps until noon every day, but attributes it to being a teenager on summer vacation. Her physical examination is within normal limits. Upon questioning she reports no trouble falling or staying asleep and goes to bed around 10 PM nightly. Which of the following is the best next step in management? A Repeat magnetic resource imaging (MRI) of the left hip. B Thyroid function studies. C Noncontrast computed tomography (CT) of the head. D Provide reassurance. E Start the patient on an antidepressant.

B. Given her older age of onset and her short stature, this patient has atypical SCFE. A patient who presents with atypical SCFE deserves an evaluation for an underlying etiology. Given this adolescent's findings of fatigue and excessive sleeping, hypothyroidism is the most likely cause. The best diagnostic test would be thyroid function studies. Depression can also result in a complaint of excessive sleeping. An evaluation for depression, rather than simply initiating antidepressant therapy, is appropriate.

45.4 You are giving a lecture to medical students about the treatment of precocious puberty and its importance in preserving height. You discuss certain population segments where treatment would be most indicated and beneficial. Which of the following patient populations would benefit most from GnRH agonist therapy? A Younger children with rapidly progressing noncentral precocious puberty B Younger children with rapidly progressing central precocious puberty C Older children with rapidly progressing noncentral precocious puberty D Older children with rapidly progressing central precocious puberty E Younger boys with slowly progressing central precocious puberty

B. GnRH agonists are beneficial in patients with precocious puberty that is rapidly progressing. The younger the onset of precocious puberty, the greater the loss to final height a child will experience. Once the epiphyseal plates close, no additional growth potential exists. GnRH agonist shows no benefit in noncentral precocious puberty because the pathology involves an excess of exogenous sex hormone not affected by the hypothalamic-pituitary-gonadal axis.

45.3 A 2-year-old boy presents to the office with his father due to concerns over the appearance of his son's genitals. He reports that over the last 2 months he has noticed his penis has gotten longer, frequent erections, and what appears to be a few dark strands of hair. On examination, you confirm that the phallus is long for stated age, pubic hair that is curly, and testicles that are less than 2.5 cm. His father denies any past medical history for his son and even states that he had imaging of his head during an ER visit for a fall 2 weeks prior which was normal. When you question the father about his onset of puberty, he claims to have noticed changes at 14 years old. He denies any other family history of illness. The child lives with his mother and father. Medications in the home include maternal intake of daily multivitamins and paternal gel for decreased libido. Which of the following statements is likely an explanation for this child's presentation? A Noncentral precocious puberty hypothalamic-pituitary-gonadal axis early activation B Noncentral precocious puberty from exogenous source of sex hormone C Central precocious puberty hypothalamic-pituitary-gonadal axis early activation D Central precocious puberty from exogenous source of sex hormone E Incomplete precocious puberty based on history and early thelarche

B. Noncentral precocious is due to exogenous sources of sex hormones. This may be from a tumor or from the use of medications. In addition, the testicle size is greater than 2.5 cm in central precocious puberty. This question highlights a black box warning regarding the use of topical testosterone. Multiple case reports of males using testosterone gel and direct skin exposure resulting in virilization of close contacts including children have been noted. This virilization usually regresses after exposure to testosterone is withdrawn.

17.4 You are seeing a 4-year-old boy with moderate diplegic CP for his well-child visit. He is nonverbal. How will your management of his visit differ from that of a child without CP? A You would not need to perform a hearing test because he is unable to respond. B You will ask which therapies he receives, who provides them, the frequency and goals. C You would not administer any vaccines. D You would ask his parents about any behavior that would suggest a seizure. E You would not perform a developmental assessment.

B. Optimal treatment for CP is multidisciplinary, and it is important to insure the child is receiving the services that are needed for ongoing treatment of his condition. Even though the office-based hearing evaluation cannot be done for him because it requires verbal participation, his hearing still needs to be assessed; a hearing test that does not require the patient to give a verbal response should be ordered. He should receive the recommended vaccines for his age. Asking about behavior that suggests a seizure is part of the review of systems for all children. Although his development is delayed, it is still important to assess in order to identify the age level that he is at for each area of development.

6.3 An 18-day-old male infant is brought to the emergency department by his mother with an 8-hour history of poor feeding and decreased wet diapers. On examination, he is afebrile, cries during venipuncture, urine catheterization, and lumbar puncture but otherwise sleeps through the examination. The remainder of his examination, including the skin, is normal. His CSF shows 78 white blood cells, 80% lymphocytes, 40 red blood cells, protein 130 mg/dL, and no bacteria on Gram stain. The mother has a cold sore at her lower lip. Which of the following would not be further part of your management? A Consult ophthalmology to assess for chorioretinitis. B HSV IgG and IgM titers. C Surface swab for HSV culture. D Complete blood cell count (CBC), ALT, and AST. E CSF for HSV culture and PCR.

B. Serology is not useful for the diagnosis of neonatal herpes infection. Neonatal herpes infection in any presentation should prompt an ophthalmology examination because topical ophthalmic antiviral therapy will be needed in addition to intravenous acyclovir if chorioretinitis or keratitis is present. A surface swab and CSF for HSV culture along with CSF HSV PCR are key steps in diagnosing neonatal HSV infection. Abnormalities in the CBC, ALT, or AST suggest that he has disseminated disease.

16.2 Three days after beginning oral amoxicillin therapy for otitis media (OM), a 4-year-old boy has continued fever, ear pain, and swelling with redness behind his ear. His ear lobe is pushed superiorly and laterally. He seems to be doing well otherwise. Which of the following is the most appropriate course of action? A Change to oral amoxicillin-clavulanate B Myringotomy and parenteral antibiotics C Nuclear scan of the head D Topical steroids E Tympanocentesis

B. The child has mastoiditis, a clinical diagnosis that can require computed tomography scan confirmation. Treatment includes myringotomy, fluid culture, and parenteral antibiotics. Surgical drainage of the mastoid air cells may be needed if improvement is not seen in 24 to 48 hours.

21.3 You are going to counsel parents of a newborn about prevention of SIDS. Which of the following statements about ways to reduce SIDS is accurate? A Infants should sleep in the same bed as the parent or on their chest so they can be closely monitored for apnea. B Infants should sleep on their back on a firm mattress with no accompanying soft bedding or objects, including no devices advertised to maintain the sleep position. C Pacifiers should be avoided because they can obstruct the baby's airflow during respiration. D Keep the infant dressed in several layers and covered with a heavy blanket. E Infants should be given acetaminophen before their scheduled vaccines in order to prevent an undetected febrile seizure and resulting SIDS.

B. The decline in SIDS has been attributed to the change in sleep position emphasizing infants be placed on their back. Other protective factors include pacifier use and vaccination. Risk factors associated with SIDS are bed sharing, prone sleep position, and overheating.

51.3 A 13-year-old girl is brought to the emergency room for altered mental status. She is not oriented to place or time and perseverates in talking about "demons" that are chasing her. The parents report she has been withdrawn for the past month, sleeping a lot, and exhibiting anorexia and compulsive behavior such as washing her hands multiple times a day. She is afebrile. Her urine drug screen and rapid HIV test are negative. Her CBC shows lymphopenia with absolute lymphocyte count of 400/mm3. Chest radiography shows bilateral pleural effusions and an enlarged heart. Electrocardiogram shows diffuse ST elevation in all leads consistent with pericarditis. What is the most appropriate next step in management? A Rapid strep test B Anti-ds DNA C Hematology consult D Psychiatry consult E Broad-spectrum intravenous antibiotics

B. The patient has three criteria of SLE: lymphopenia, serositis (pleuritis and pericarditis), and psychosis. Further specific testing for SLE should be done.

40.1 A 16-year-old boy has a 2-week history of subjective fever, sore throat, and swollen and tender lymph nodes in the neck and groin. He admits to being sexually active, including oral sex, with a male partner over the past month. They do not use condoms. On physical examination, he is afebrile, with cervical and inguinal lymphadenopathy and a nonexudative pharyngitis. HIV ELISA done in the ER was negative. Which of the following is the next best step in your evaluation? A Lymph node biopsy B HIV RNA PCR C CD4 cell count D Herpes simplex virus-1 IgG E Rapid plasma reagin (RPR)

B. This adolescent presents with nonspecific findings of a viral-like illness. Sexual activity, a male partner, and absent barrier protection are risk factors for acquiring HIV. Because an early negative HIV ELISA is possible, HIV RNA PCR to detect replicating virus is a reasonable consideration. CD4 count is premature, routine nodal biopsy may not provide a diagnosis, and herpes or syphilis is not consistent with the clinical scenario. Whether symptomatic or not, selective and judicious testing for sexually transmitted diseases (STDs) is an important consideration in all sexually active adolescents.

19.3A mother brings to the clinic her 4-year-old son who began complaining of right knee pain 2 weeks ago, is limping slightly, is fatigued, and has had a fever to 100.4°F (38°C). Which of the following laboratory tests is most important? A Antinuclear antibodies B Complete blood count (CBC) with differential and platelets C Epstein-Barr virus titer D Rheumatoid factor E Sedimentation rate

B. This child has symptoms consistent with both juvenile idiopathic arthritis (JIA) and leukemia. The CBC with differential and platelets is the best initial screening test. The leukocyte and platelet counts are normal to increased in JIA, and no blast cells are present. Frequently, blast cells are found on the peripheral smear with ALL. The child in the question ultimately may require a bone marrow aspiration.

38.4 A 9-month-old girl is fussy, appears to have pain on palpation of the right leg, and has bluish sclerae. Radiographs reveal a right femur fracture. Her parents deny any severe trauma but report she had multiple fractures as a child. Family history is also likely to include which of the following? A Blindness B Short stature C Tall stature D Renal disease E Aortic aneurysm

B. This infant has features of osteogenesis imperfecta, an autosomal dominant genetic disorder most often caused by point mutations of COL1A1 or COL1A2 genes. Features include long bone fractures and vertebral injury with minimal trauma, short stature, deafness, and blue sclerae. Four main types exist: type 1 is mild; type 2 is lethal (in utero or shortly thereafter); type 3 is the most severe; and type 4 is moderately severe. The types 5 to 7 do not involve mutations of type 1 collagen.

38.1 A 3-month-old boy is seen by the pediatrician with swelling of his right upper arm without history of trauma. On physical examination the infant has tenderness of the arm without erythema and is noted to have right-sided scalp swelling and multiple small bruises on the torso. The infant is otherwise well appearing and has an interactive smile. Which of the following is the best next step in management? A Obtain a radiograph of the right elbow. B Order a skeletal survey. C Place the right arm in a sling. D Order a prothrombin time (PT) and partial thromboplastin time (PTT). E Apply traction to the forearm while increasing the degree of pronation.

B. This infant has what appears to be multiple injuries without a history of trauma. This is concerning for possible inflicted injury. A skeletal survey must be performed.

33.2 A 1-week-old infant presents with a right midquadrant abdominal mass and decreased urinary output. There has been no temperature lability, irritability, or abnormal stooling or urine appearance. Which of the following tests would be most helpful in determining the etiology of this infant's abdominal mass? A Complete blood count B Abdominal ultrasound C Urinary catecholamines D Abdominal computed tomography (CT) E Barium enema

B. This infant most likely has a urinary tract obstruction. In the newborn, a palpable abdominal mass is commonly a hydronephrotic or multicystic dysplastic kidney, and typically can be easily identified by ultrasound (differentiates solid versus cystic masses, easily attainable in infants, involves no radiation exposure).

36.4 A 5-year-old boy presents to the emergency room with his mother with a complaint of left shoulder pain and fever for a week. Vitals include temperature of 102.3°F (39.1°C), heart rate 95 beats/min, respiratory rate 24 breaths/min, and blood pressure 96/54 mm Hg. On physical examination, the left shoulder is warm and tender to palpation, and has decreased range of motion. Laboratory test results were drawn and were positive for white blood cell (WBC) count of 17,000/μL, erythrocyte sedimentation rate (ESR) of 46 mm/h, and C-reactive protein (CRP) of 18 mg/L. What is the diagnosis? A Transient synovitis B Septic arthritis C Humerus fracture D Shoulder dislocation

B. This patient has a fever, high WBC count, elevated ESR and CRP, and physical examination positive for a warm and tender joint. There is an infectious process occurring. To distinguish septic arthritis from transient synovitis, the Kocher criteria are used: fever, refusal to move joint or bear weight, WBC greater than 12,000/μL, and ESR greater than 40 mm/h. This is an orthopedic emergency. The next step is to aspirate the joint and send laboratory tests on the joint aspirate.

13.2 A 14-year-old boy with known SCD has been admitted for vaso-occlusive crisis. He has been receiving IV fluids and IV narcotics for the last 2 days. He seemed to be getting better, but the nurse has just called and stated his temperature is now 101.8°F. On examination, you find his respiratory rate to be 32 breaths/min, and his oxygen saturation is 90%. What is the next step in management? A Order spiral CT. B Initiate antibiotics and provide supplemental oxygen via nasal cannula. C Obtain STAT hemoglobin level. D Initiate incentive spirometry. E Administer another dose of narcotics.

B. This patient has been admitted for vaso-occlusive crisis and is at high risk of developing acute chest syndrome (ACS). Given that he currently has a fever, tachypnea, and decreased oxygensaturation, he meets ACS criteria. Prompt initiation of antibiotics and supplemental oxygen is imperative in preventing deadly complications of ACS. Although pulmonary embolism is in the differential for this patient and an acute drop in hemoglobin might cause tachypnea, initiation of antibiotics should be your first step in management. Incentive spirometry is an important part of the prevention of ACS, but antibiotics are more important in its treatment. Although increased pain may result in tachypnea, it would not usually cause a decrease in the patient's oxygen saturation.

32.2 An otherwise healthy 8-year-old girl, with normal growth and development and no recurring rash history, arrives at your clinic complaining about very itchy lesions on her chest, abdomen, and arms. The dermatitis started with a few red, raised areas on her chest 3 days ago, and then quickly spread. They appear to come and go in different areas. Physical examination reveals many ovoid, red, and raised eruptions on her torso and arms. Some are confluent, many are warm to touch or tender, and a few are weepy, crusty, or scaly with excoriations nearby. She is afebrile and the rest of her examination is unremarkable. Which of the following is the most important medical historical clue for achieving an accurate diagnosis? A Her infant brother has severe eczema. B She just completed an unidentified oral antibiotic course for her first bladder infection. C Her mother has recurring "spider bites" that often require drainage and antibiotics. D She had many ear infections as a toddler, and a few required repeat antibiotic courses. E Her father had allergies and asthma as a child.

B. This patient's symptoms and rash are most consistent with acute urticaria and possible bacterial superinfection from scratching. The most likely etiology for her hives is her recent antibiotic use. Her family's atopic history is important, but her current rash and her past benign skin history are inconsistent with eczema. Recurring ear infections in the context of a patient with no failure to thrive or serious, difficult-to-eradicate infections make immune system dysfunction and associated dermatitides less likely. Her mother's "spider bites," requiring drainage and antibiotics, infer possible colonization and infection with methicillin-resistant Staphylococcus aureus. The patient herself may be an asymptomatic nasal or skin carrier and have seeded excoriations when scratching.

19.1 A mother brings her 3-year-old son with Down syndrome to the clinic because his gums have been bleeding for 1 week. She reports that he has been less energetic than usual. Examination reveals that the child has an oral temperature of 100°F (37.8°C), pallor, splenomegaly, gingival bleeding, and bruises on the lower extremities. Which of the following is most likely? A Aplastic anemia B Idiopathic thrombocytopenic purpura (ITP) C Leukemia D Leukemoid reaction E Megaloblastic anemia

C. A high susceptibility to leukemia is associated with certain heritable diseases (Klinefelter syndrome, Bloom syndrome, Fanconi syndrome, ataxia telangiectasia, neurofibromatosis) and chromosomal disorders such as Down syndrome. Children with Down syndrome have a 10- to 15-fold increased risk for developing leukemia and should have routine screening performed at well-child checks. Siblings of an acute lymphoblastic leukemia (ALL) patient have a two- to fourfold increased risk for ALL, and the monozygotic twin of a child who develops ALL in the first year of life has a more than 70% chance of also developing ALL. A few cases of ALL are associated with p53 gene aberrations. Overall, these genetic links account for a small number of total ALL cases.

15.3 A 14-year-old girl reports she has intermittent periods of blood in her stools that lasts 2 days. Once the bleeding stops, it does not recur for about 2 weeks. She denies fever, abdominal pain, and diarrhea. She affirms that she has constipation. On physical examination, you see an anal fissure. You counsel her on how to take PEG 3350 and how to apply petrolatum to the site of the fissure. What else should you discuss with her in order to prevent a recurrence of the fissure? A She should use a suppository if she feels constipated again. B She should increase her dietary fiber intake. C She needs to return to the clinic for further evaluation if the fissure recurs despite resolution of the constipation. D She should decrease the number of times she goes to defecate. E She should apply triple antibiotic ointment to the fissure.

C. A suppository, increased fiber, and stool withholding will worsen the stool bulk and are likely to cause a fissure. The etiology for her fissure formation is not infectious. If she no longer has constipation but the fissure recurs, then Crohn disease may be the cause and she should have further evaluation.

12.3 A 12-month-old patient is seen in the clinic. His mother reports that she exclusively breast-fed the child until he was 6 months of age. He still breast-feeds approximately four to five times daily but the mother reports he also drinks 2 to 3 oz of water daily, 8 to 10 oz of juice daily, and has been doing well eating table foods. What dosing of vitamin D would be the most appropriate for this child? A 200 IU daily B 400 IU daily C 600 IU daily D 800 IU daily E 1000 IU daily

C. All infants younger than 12 months should receive 400 IU daily beginning within the first few days after birth. Healthy children 1 to 18 years of age should receive 600 IU daily. Children with liver disease, kidney disease, or malabsorption (ie, secondary to cystic fibrosis, celiac disease, inflammatory bowel disease [IBD], etc.) as well as premature infants need even higher doses to account for the limitations in vitamin D metabolism or absorption brought on by these health issues.

16.1 A 1-year-old boy presents with fever, ear pain, and purulent discharge from both eyes. On examination, bilateral erythematous, bulging tympanic membranes (TMs), and purulent conjunctivitis are noted. Based on his symptoms and examination, amoxicillin-clavulanate is prescribed. The mother asks what she could do to prevent future ear infections. Which of the following is NOT one of the current recommendations to reduce the incidence of otitis media? A Pneumococcal vaccine B Influenza vaccine C Xylitol D Eliminating exposure to tobacco smoke E Breast-feeding

C. Although some research shows xylitol use can reduce AOM, compliance issues make it an ineffective therapy. With the pneumococcal vaccine, a 2009 Cochrane review showed the overall incidence of AOM has reduced by 6% to 7%. Because as many as two-thirds of children who have influenza have AOM, studies have demonstrated the influenza vaccine has a 30% to 55% efficacy of preventing AOM during respiratory season. Breast-feeding for at least 4 to 6 months reduces episodes of AOM and recurrent AOM. In addition to other benefits, elimination of passive tobacco smoke exposure has been linked to a reduction of AOM in infancy.

10.3 A 2-year-old boy had been slightly less than the 50th percentile for weight, height, and head circumference, but in the last 6 months he has fallen to slightly less than the 25th percentile for weight. The pregnancy was normal, his development is as expected, and the family reports no psychosocial problems. The mother says that he is now a finicky eater (wants only macaroni and cheese at all meals), but she insists that he eat a variety of foods. The meals are marked by much frustration for everyone. His examination is normal. Which of the following is the best next step in his care? A Sweat chloride testing B Ophthalmologic examination for retinal hemorrhages C Reassurance and counseling for family about normal childhood development D Testing of stool for parasites E Magnetic resonance imaging (MRI) of the brain

C. Between 18 and 30 months of age, children often become "picky eaters." Their growth rate can plateau, and the period can be distressing for families. Of note, this patient's growth decline has not crossed two major growth percentiles to define FTT. Counseling parents how to provide optimal nutrition, avoid "force-feeding," and avoid providing snacks is usually effective. Close follow-up is required.

15.1 A 5-year-old boy is brought to clinic by his mother for evaluation of blood in his stool. Yesterday, the boy told her mother that he saw "red stuff" on his stool and when the mother visualized the stool, it appeared to have "threads of blood" over the surface. The child denies any blood with wiping. Which of the following would indicate that an anal fissure is NOT the cause of the rectal bleeding? A The boy has a 1-month history of encopresis. B The child's height and weight are at the 75th percentile for his age. C The boy's conjunctiva, gingiva, and nail beds are pale. D The child affirms that it has been painful to wipe after stooling. E The mother reports that since starting kindergarten, her son prefers to eat the school lunch, consisting of pizza and chocolate milk, rather than a homemade sandwich with water.

C. Bleeding from an anal fissure is not chronic and the amount of hematochezia is small. A hemoglobin that is low enough to cause pallor of the mucosal membranes is not produced from an anal fissure.

8.3 A term male is born vaginally to a 22-year-old primigravida woman; the pregnancy was uncomplicated. Just prior to delivery, fetal bradycardia was noted, and at delivery thick meconium is found. The infant has hypotonia and bradycardia. Which of the following is the first step in resuscitation? A Administration of epinephrine through endotracheal tube B Bag-mask ventilation C Endotracheal intubation with direct suction D Oxygen delivered by cannula in close proximity to the nares E Tracheostomy

C. Endotracheal intubation with direct suctioning of meconium below the vocal cords is performed in a depressed infant with thick meconium noted at delivery. Bag-mask ventilation or endotracheal intubation without suction may increase the volume of meconium aspirated. A vigorous infant with a heart rate greater than 100 beats/min, strong respirations, and good muscle tone with meconium-stained need not be suctioned immediately after birth.

19.4 Two weeks after a viral syndrome, a 2 year old develops bruising and generalized petechiae that is more prominent over the legs. He has neither hepatosplenomegaly nor lymph node enlargement. Laboratory testing reveals a normal hemoglobin, hematocrit, and white blood cell count and differential. The platelet count is 15,000/mm3. Which of the following is the most likely diagnosis? A Acute lymphoblastic leukemia (ALL) B Aplastic anemia C Immune thrombocytopenic purpura (ITP) D Thrombotic thrombocytopenic purpura (TTP) E von Willebrand disease (VWD)

C. Immune (or idiopathic) thrombocytopenic purpura (ITP) is common in children. In most cases, a preceding viral infection can be documented. The platelet count frequently is less than 20,000/mm3, but other laboratory test results are normal, including the bone marrow aspiration (which may show an increase in megakaryocytes). Treatment consists of observation or possibly intravenous immunoglobulin (IVIG), intravenous anti-D (in Rh-positive patients), immunosuppressives, or steroids. The history must be reviewed for other possible causes of thrombocytopenia, including recent MMR vaccination, drug ingestion, and human immunodeficiency virus.

17.2 A 4-year-old child with CP comes to your clinic for the first time for a routine visit. He walks with the help of leg braces and a walker, and his speech is slurred and limited to short phrases. He has never been hospitalized, and he does not have swallowing problems. He began walking at the age of 2.5 years, and he is unable to take off his clothes and use the toilet without help. On examination, you find that the boy has only minimally increased tone in the upper extremities but good fine motor coordination; he has significantly increased tone and deep tendon reflexes in the lower extremities. How would you categorize this child's CP? A Mild, diplegic B Mild, hemiplegic C Moderate, diplegic D Moderate, quadriplegic E Severe, diplegic

C. In diplegia all four extremities are affected, with greater impairment of the lower extremities. Because most children walk by the age of 14 months, this child's motor quotient is 14 months/30 months = 0.47, which classifies him as moderately impaired.

2.1 A 36-week gestation infant is delivered via cesarean section because of macrosomia and fetal distress. The mother has class D pregestational diabetes (insulin dependent, with vascular disease); her hemoglobin A1c is 15% (normal 7%). This infant is at risk for hypocalcemia, cardiomyopathy, polycythemia, and which of the following? A Congenital dislocated hip B Dacryostenosis C Respiratory distress syndrome D Hyperglycemia E Pneumothorax

C. Infants born to mothers with poorly controlled diabetes are at risk for respiratory distress syndrome (surfactant deficiency) even at near-term gestational ages. The maternal hyperglycemia stimulates fetal hyperinsulinism, which has an antagonistic effect on surfactant production.

18.4 A 3-month-old infant is admitted to the hospital with her third episode of lobar pneumonia and wheezing. Findings that would increase your suspicions for CF and prompt sweat chloride testing include all of the following EXCEPT: A Hyponatremia, hypochloremia, and metabolic alkalosis B Failure to thrive C Lymphocytosis D Digital clubbing E Oily appearing stools

C. Infants with CF will lose excess amounts of sodium chloride in their sweat resulting in a hyponatremic, hypochloremic metabolic alkalosis. Malabsorption of fats and protein due to pancreatic exocrine insufficiency usually presents as steatorrhea and is a major cause of morbidity for patients with CF, resulting in failure to thrive. Lymphocytosis can occur with viral infections or pertussis but is not a hallmark of CF.

36.2 The mother of an anxious 4-year-old girl presents to the pediatrician's office with a chief complaint of the patient not moving her right arm. The mother gives a history of the patient's father swinging her by her arms earlier in the day. The patient suddenly yelled out in pain while the father was swinging her. She clutched her arm close to her abdomen and would not allow anyone to touch it. On physical examination, the girl resists movement of her right arm. The affected arm is without tenderness to palpation or swelling. The girl has not been febrile. The doctor attempts to supinate the affected arm with the elbow in flexed position. She attempts three times but fails to hear a click and the child is still in pain and holding her arm close to her body. Which of the following is the most appropriate next step in her management? A Reattempt reduction maneuvers until a click is heard and patient can move her arm. B Aspirate the joint fluid for suspected infection. C Radiograph of the right arm. D MRI of the right arm. E Inform Child Protective Services.

C. Initial management includes two to four attempts about 15 minutes apart in an attempt to reduce the annular ligament. Should the expected improvement not be seen, radiographs of the affected extremity are indicated.

31.4 A 32-year-old G2 P1 pregnant woman who is at 14 weeks of gestation and is a known carrier of Duchenne muscular dystrophy (DMD) asks a genetic counselor about the possibilities that her unborn child will inherit DMD. The father is healthy. Of the following statements, the genetic counselor would be most accurate in saying: A If she had a daughter, she has a 100% chance of having DMD. B If she had a daughter, she has a 100% chance of being a carrier of DMD. C If she had a son, he has a 50% chance of having DMD. D If she had a son, he has a 50% chance of being a carrier of DMD. E If she had a son, he has no chance of having DMD.

C. Males have one copy of the X chromosome, while females have two copies of the X chromosome. If mother is a carrier, then she has one copy with a mutation and one functional copy. Therefore, she has a 50% chance of passing the copy with a mutation to her daughters or sons. Daughters who receive the copy with a mutation can only be carriers, whereas sons who receive the copy with a mutation can only have DMD. Therefore, daughters have a 50% chance of being carriers and sons have a 50% chance of having DMD.

34.2 An ill-appearing 7-day-old boy presents with 72 hours of bilious vomiting, abdominal pain, and abdominal distension. Which of the following is the most helpful to establish a diagnosis? A Order an abdominal ultrasonography. B Order a computed tomography scan of the abdomen. C Order an upper gastrointestinal (GI) contrast series. D Order a barium enema. E Order a chest radiograph.

C. Order an upper GI contrast series. Prior to emergent surgical intervention, the initial management of patients with malrotation and volvulus includes appropriate evaluation of fluid status because patients may have significant fluid loss with electrolyte abnormalities. In the ill-appearing infant, placement of a nasogastric tube to aid gastrointestinal decompression, and initiation of parenteral antibiotic, in order to address potential sepsis are indicated.

51.1 Which antibody is NOT part of the diagnostic criteria for systemic lupus erythematosus (SLE)? A Anti-Sm B Rapid plasma reagin (RPR) C Rheumatoid factor (RF) D Anticardiolipin E Lupus anticoagulant

C. Rheumatoid factor is not part of the diagnostic criteria for SLE.

12.1 An 18-month-old infant is seen in the pediatrician's office as a new patient for a well-child visit having moved from Minnesota to Texas. A comparison of new to previous growth parameters suggest he has dropped to less than fifth percentile for height, weight, and head circumference. Physical examination findings are significant for lower-extremity bowing, wrist enlargement, frontal bossing, and some hypotonia. The mother is concerned because he does not pull to stand nor walking. She reports that he was exclusively breast-fed until he was 9 to 10 months old, currently drinks about 24 oz of whole milk daily, and has never taken medications or dietary supplements. The mother also has a question about whether he needs allergy testing given his history of frequent upper respiratory infections. Which of the following laboratory findings would be expected? A Low calcium, normal phosphorus, elevated alkaline phosphatase, elevated parathyroid hormone (PTH) B Low calcium, low phosphorus, low alkaline phosphatase, elevated PTH C Low calcium, low phosphorus, elevated alkaline phosphatase, elevated PTH D Low calcium, high phosphorus, elevated alkaline phosphatase, elevated PTH E Low calcium, low phosphorus, low alkaline phosphatase, low PTH

C. Rickets secondary to vitamin D deficiency frequently presents with low to normal serum calcium (depending on the duration of the disease), elevated serum PTH (to compensate for the calcium deficiency), elevated serum alkaline phosphatase, and low serum phosphorous. PTH mobilizes calcium from the bone and subsequently increases renal phosphate loss.

40.3 A 6-month-old girl is seen after an emergency room visit for decreased intake, emesis, and watery diarrhea for the past 3 days. She was diagnosed yesterday with "stomach flu" and given IV fluids. She is doing better today with improved intake and resolution of her emesis and diarrhea. The father is concerned about her thrush since birth (despite multiple courses of an oral antifungal), and that she has been hospitalized twice for pneumonia over the past 4 months. Her weight has dropped from the 50th percentile on her 4-month visit to the 5th percentile today. She has no findings consistent with dehydration, but she does appear to have some extremity muscle wasting. Her examination is remarkable for buccal mucosal exudates and hyperactive bowel sounds. Vital signs and the remainder of her examination are normal. You suspect severe combined immunodeficiency (SCID). Which of the following is consistent with the diagnosis? A Autosomal dominant inheritance B Persistent lymphocytosis C Defective cellular immunity D Normal vaccine immune response E No curative therapy

C. Severe combined immunodeficiency (SCID) is an autosomal recessive or X-linked disorder of both humoral and cellular immunity. Serum immunoglobulins and T cells are often markedly diminished or absent. Thymic dysgenesis is also seen. Recurring cutaneous, gastrointestinal, or pulmonary infections occur with opportunistic organisms such as cytomegalovirus (CMV) and Pneumocystis pneumonia (PCP). Death typically occurs in the first 12 to 24 months of life unless bone marrow transplantation is performed.

14.2 A 2-year-old girl has increased work of breathing. Her father notes she has had cough and subjective fever over the past 3 days. She has been complaining that her "belly hurts" and has experienced one episode of posttussive emesis, but no diarrhea. Her immunizations are current, and she is otherwise healthy. Her temperature is 102°F (38.9°C). She is somnolent but easily aroused. Respirations are 28 breaths/min, and her examination is remarkable for decreased breath sounds at the left base posteriorly with prominent crackles. Which of the following acute interventions is the next best step in your evaluation? A Blood culture B Chest radiography C Pulse oximetry D Sputum culture E Viral nasal swab

C. Tachypnea and lethargy are prominent in this patient with clinical pneumonia. Pulse oximetry should urgently be performed to ascertain whether oxygen is required. Sputum culturing is reasonable for an older patient who can produce sputum, but an adequate and diagnostically useful specimen can only be obtained from a 2-year-old by endotracheal aspirate or bronchoscopy. In this otherwise healthy toddler for whom concerns for atypical pneumonia are high, invasive maneuvers are not indicated. Viruses (respiratory syncytial virus [RSV] and adenovirus) are prominent at this age; one might consider performing a nasal swab viral PCR. Abdominal pain, as noted in this question, can be seen as a presenting symptom in pneumonia, probably because of irritation of the diaphragm by pulmonary infection.

34.1 Malrotation with volvulus is most likely to be present in which of the following patients? A A healthy 15-month-old with severe paroxysmal abdominal pain and vomiting B A 15-year-old sexually active girl with lower abdominal pain C A 3-day-old term infant with bilious emesis, lethargy, and abdominal distension D A 4-day-old premature baby (33-week gestation) who has recently started nasogastric feeds; he now has abdominal distention, bloody stools, and thrombocytopenia E A 7-year-old non-toxic-appearing girl with abdominal pain, vomiting, fever, and diarrhea

C. The 3-day-old term infant with bilious emesis and abdominal distension has classic presenting features of malrotation with volvulus. The 15-month-old child with paroxysmal abdominal pain is most likely to have intussusception. The adolescent girl is evaluated for ectopic pregnancy, pelvic inflammatory disease, appendicitis, ovarian torsion, and ruptured ovarian cyst. The premature infant might have necrotizing enterocolitis, whereas the 7-year-old girl most likely has gastroenteritis.

45.2 A 7-year-old girl presents to your office with her mother noting pubic hair growth over the last month. The girl also has started to develop breast buds. Growth and development are normal. Maternal history is remarkable for pubarche at age 11 years. Past medical history and family history are otherwise unremarkable. Your physical examination reveals genitalia and breast Tanner stage 3. Which of the following laboratory findings is most likely to be found? A Decreased estradiol, luteinizing hormone (LH) detectable, normal bone age for age B Decreased estradiol, LH undetectable, normal bone age for age C Elevated estradiol, LH detectable, advanced bone age for age D Elevated estradiol, LH detectable, normal bone age for age E Elevated estradiol, LH undetectable, advanced bone age for age

C. The child likely has precious puberty. With true precocious puberty hormonal levels mirror the physical appearance of the child (increased estrogen and detectable LH) because a premature activation of the hypothalamic pituitary gonadal axis has occurred. The bone age would also reflect these changes and appear older on wrist x-ray than her chronologic age.

10.1 Parents bring their 6-month-old son to see you. He is symmetrically less than the fifth percentile for height, weight, and head circumference on routine growth curves. He was born at 30 weeks of gestation and weighed 1000 g. He was a planned pregnancy, and his mother's prenatal course was uneventful until an automobile accident initiated the labor. He was ventilated for 3 days in the intensive care unit (ICU), but otherwise did well without ongoing problems. He was discharged at 8 weeks of life. Which of the following is the mostly likely explanation for his small size? A Chromosomal abnormality B Protein-calorie malnutrition C Normal ex-preemie infant growth D Intestinal malabsorption E Congenital hypothyroidism

C. The expected weight versus age must be modified for a preterm infant. Similarly, growth for children with Down or Turner syndrome varies from that for other children. Thus, use of an appropriate growth curve is paramount. For the child in the question, weight gain should follow or exceed that of term infants. For this premature infant, when his parameters are plotted on a "preemie growth chart," normal growth is revealed.

47.1 A 7-year-old boy is seen in the pediatrician's office because his mother has noticed him limping for the past few days. She has given him acetaminophen with only mild relief of pain. His temperature is 101°F (38°C), HR 120 beats/min, BP 100/70 mm Hg, and his weight is at the 97th percentile for age. On examination, his right knee is red, warm, and swollen. Range of motion testing is limited because the patient is guarding and begins crying. What of the following is the best next step in management? A Send the patient for an x-ray of his knee and hip joint and ask them to return the same day. B Provide reassurance. The mainstay of therapy is rest, ice, compression, and elevation. C Immediately admit the patient to the hospital for IV antibiotics. D Obtain an erythrocyte sedimentation rate (ESR) and C-reactive protein (CRP) and ensure early follow-up the following day. E Send a referral to orthopedic surgery.

C. The patient likely has septic arthritis of his right knee and needs immediate hospital admission for intravenous antibiotics. His evaluation would include complete blood count, an ESR, a CRP, and blood cultures. In addition, he requires a joint aspiration for Gram stain and culture. Immediate inpatient or emergency center orthopedic surgery evaluation for the possibility of surgical drainage and wash out of his knee joint is appropriate. Septic joint is an emergency because the joint can be destroyed within hours. Depending on his hospital course, the patient may require long-term IV antibiotics or may be transitioned to oral therapy. SCFE, developmental dysplasia of the hip (DDH), or Legg-Calve-Perthes do not present with fever. Moreover, erythema and warmth are not typical features of any of these latter conditions.

18.2 A 7-year-old girl is admitted to the hospital in respiratory distress due to pneumonia. This is her third admission in the past 6 months. At this time you are suspecting cystic fibrosis (CF) and order a sputum culture. Which organism would be most consistent with a diagnosis of cystic fibrosis? A Streptococcus pneumoniae B Mycobacterium tuberculosis C Pseudomonas aeruginosa D Bacillus cereus E Haemophilus influenzae

C. The presence of P aeruginosa on a sputum sample strongly suggests the diagnosis of CF. Patients with CF have a high prevalence of colonization with P aeruginosa, Staphylococcus aureus, and Burkholderia cepacia. The innate defenses of the airway epithelium cells of CF patients may be compromised, making them unable to fight these organisms.

14.3 You are evaluating a previously healthy 8-year-old boy with subjective fever, sore throat, and cough over the past week. There has been no rhinorrhea, emesis, or diarrhea, and his appetite is unchanged. According to your clinic records, his immunizations are current and his weight was at the 25th percentile on his examination 6 months ago. Today, he is noted at the 10th percentile for weight. He is afebrile, with clear nares and posterior oropharynx, and a normal respiratory effort. He has bilateral cervical and right supraclavicular lymphadenopathy. Chest auscultation is notable for diminished breath sounds at the left base. Beyond obtaining a chest radiograph, which of the following is the best next step in your evaluation? A Rapid strep throat swab B Viral nasal swab C Purified protein derivative (PPD) placement D Lymph node biopsy E Bordetella pertussis direct fluorescent antibody testing

C. The scenario is typical for pediatric tuberculosis. Neck and perihilar or mediastinal lymphadenopathy and pulmonary or extrapulmonary manifestations can occur, with miliary disease and meningitis more common in infants and younger children. Fever, weight loss, and lower respiratory tract signs and symptoms (possible left pleural effusion in this patient) are archetypal TB findings. A PPD should be placed or IGRA drawn, and consideration given to hospitalizing this patient in negative pressure isolation for further evaluation beyond initial screening (pleurocentesis, bronchoalveolar lavage, gastric aspirates) and possible antituberculous treatment.

37.2 A 14-year-old adolescent girl has a rash on her arms and legs. She was diagnosed with a urinary tract infection 4 days ago, which is being treated with trimethoprim-sulfamethoxazole. She denies fever, vomiting, diarrhea, headache, and dysuria. On examination she has multiple upper- and lower- extremity petechiae. Her WBC count is 7000/mm3 and hemoglobin level is 13 mg/dL; her platelet count is 35,000/mm3. Which of the following is the best next step in management? A Send blood for antinuclear antibody (ANA). B Send a repeat urinalysis. C Discontinue the trimethoprim-sulfamethoxazole. D Obtain HIV testing. E Administer intravenous immunoglobulin.

C. The thrombocytopenia may be because of the trimethoprim-sulfamethoxazole; the medicine is discontinued and her platelet count is monitored. If thrombocytopenia continues, she may have ITP and should be followed for the development of chronic ITP. Chronic ITP occurs in older children (female predominance); it may be seen with autoimmune disease such as systemic lupus erythematosus or with chronic infections including HIV.

6.4 A 38-week gestation male infant was precipitously delivered vaginally in the emergency department. Weight and head circumference are less than 5th percentile, and length at 10th percentile. He has hepatosplenomegaly and a petechial rash. Notable laboratory results include platelets of 22,000/mm3 and elevated total and direct bilirubin levels. CT of the head shows bilateral intracranial calcifications with several around the basal ganglia and obstructive hydrocephalus. Ophthalmologic examination reveals chorioretinitis. What is the most likely cause of the patient's findings? A Cytomegalovirus B Rubella virus C Toxoplasmosis gondii D Herpes simplex virus E Treponema pallidum

C. The triad of hydrocephalus, intracranial calcifications, and chorioretinitis is a classic presentation for congenital toxoplasmosis. With cytomegalovirus, the intracranial calcifications would be in a periventricular distribution and the typical rash would be the blueberry muffin rash. Rubella would be expected to also have the purpuric rash along with cataracts and no CT findings. Herpes would have either vesicles or scarring. Syphilis infection would not have an abnormal CT.

2.2 A term infant weighing 4530 g is born without complication to a mother with gestational diabetes. At 12 hours of life, he appears mildly jaundiced. Vital signs are stable, he is eating well, and his blood type is the same as his mother's blood type. Which of the following serum laboratory tests are most likely to help you evaluate this infant's jaundice? A Total protein, serum albumin, and liver transaminases B Total and direct bilirubin, liver transaminases, and a hepatitis panel C Total bilirubin and a hematocrit D Total bilirubin and a glucose E Total and direct bilirubin and a calcium

C. This baby most likely has hyperbilirubinemia due to polycythemia. Total bilirubin and hematocrit should be measured to determine if phototherapy is needed. The polycythemia warrants close monitoring of the infant's hydration in order to avoid worsening the hyperviscosity.

44.2 A 16-year-old boy complains that he is the shortest boy in his class. He has a normal past medical history, and although always a bit small for age, he has really noticed that he has fallen behind his peers in the last 2 years. He is Tanner stage 3 and is at the 5th percentile for height. His father began puberty at the age of 16 and completed his growth at the age of 19; he is now 6 ft 2 in (188 cm) tall. His mother began her pubertal development at the age of 10 and had her first menstrual period at the age of 13; her height is 5 ft 4 in (163 cm). Which of the following is the single most appropriate first intervention? A Chromosomal analysis B Liver function studies C Measurement of bone age D Measurement of somatomedin C E Pediatric endocrinology referral

C. This boy likely has constitutional growth delay, similar to that of his father. Bone age would be delayed, indicating potential growth. He eventually will enter puberty, but the psychosocial ramifications of remaining shorter and appearing more immature than his peers may warrant treatment. Monthly testosterone injections "jump start" the pubertal process without altering final growth potential; a pediatric endocrinologist might be required to assist.

33.4 During a routine preventive health visit for a 3-year-old boy, you incidentally note an irregular abdominal mass involving both lower quadrants. His mother denies having noted this previously and declares her son to be generally healthy. There has been neither gastrointestinal distress nor apparent abdominal pain. Beyond the abdominal mass and pallorous conjunctivae, his vital signs and physical examination are normal. Which of the following tests would be most helpful in determining the etiology of his abdominal mass? A Abdominal radiograph B Chest radiograph C Urinary catecholamines D Complete blood count E Urine myoglobin

C. This boy's history and examination are consistent with neuroblastoma. Given the vast majority of neuroblastoma patients have elevated urinary catecholamines, a 24-hour quantitative assessment of these metabolites should be confirmatory.

44.4 A 10-year-old girl is 4 ft 10 in (147 cm) tall. Her father is 5 ft 10 in (178 cm) tall, and her mother is 5 ft 5 in (165 cm) tall. Over the past 3 years she has dropped from the 25th percentile for height to the 5th percentile. Her weight has remained stable. Radiographs show bone age is delayed. Which of the following is the most appropriate next step in her management? A Send patient to the emergency department for CT evaluation of the brain. B Obtain growth hormone level. C Obtain IGF-1 level. D Start growth hormone injections. E Provide reassurance to parents.

C. This child has evidence of possible growth hormone deficiency. Obtaining an IGF-1 level would be the most appropriate initial choice. Growth hormone deficiency is not an emergent condition but the earlier the diagnosis is made the sooner appropriate treatment can be initiated. Obtaining a growth hormone level would not be diagnostically helpful because its secretion is pulsatile. Referral to a pediatric endocrinologist would be warranted if initial screening tests confirm the suspected growth hormone deficiency.

37.3 A 7-year-old boy has a rash on his lower extremities and pain in his right knee. He has had a low-grade fever and abdominal pain, and he has felt tired. He is nontoxic appearing, but he has palpable petechiae on his lower extremities and buttocks. His right knee is mildly edematous and he can bear weight on his right leg, but complains of pain. His prothrombin time (PT), partial thromboplastin time (PTT), and platelet counts are normal. Which of the following is the best next step in management? A Begin a course of systemic corticosteroids. B Begin empiric antimicrobial therapy for sepsis. C Obtain a urinalysis and provide supportive care. D Perform aspiration of the synovial fluid in his right knee. E Administer intravenous immunoglobulin.

C. This child has signs and symptoms of Henoch-Schönlein purpura (HSP), a vasculitis of the small vessels with renal, gastrointestinal, joint, and dermatologic involvement. Initial therapy consists of hydration and pain control. With renal involvement, urinalysis reveals red blood cells (RBCs), WBCs, casts, or protein. Gastrointestinal complications include hemorrhage, obstruction, and intussusception; abdominal pain requires careful evaluation.

8.4 A newborn female is delivered by C-section to a 23-year-old mother after 29 weeks of gestation. She has poor respiratory effort at time of delivery with cyanosis, requiring resuscitation and eventually intubated. On examination in the delivery room, the infant continues to have subcostal retractions and is difficult to ventilate. What is the next BEST step in management? A Obtain a chest x-ray. B Administer albuterol. C Administer surfactant. D Closely monitor clinically. E Obtain an echocardiogram (ECHO).

C. This infant born prematurely likely has respiratory distress syndrome from surfactant deficiency. Difficulty ventilating arises due to collapsed alveoli. Introduction of surfactant as a treatment in delivery rooms decreases alveoli surface tension thus improving ventilation and has increased premature infants survival. A need for ECHO and chest imaging maybe required if symptoms persist.

32.4 An 8-month-old child has refractory eczema that was first noticed at 2 months of age. His past medical history reveals multiple episodes of otitis media and pneumonia, and he has now developed severe nose bleeds. You suspect a primary immunodeficiency. Which of the following is the next best test in your evaluation? A Sweat chloride test B Chest computed tomography (CT) C Complete blood count (CBC) D CD4 cell count E Referral to ear, nose, and throat (ENT) for nasal endoscopy

C. This patient most likely has Wiskott-Aldrich syndrome, an X-linked condition with recurrent infections, thrombocytopenia, and eczema. Infections and bleeding usually are noted in the first 6 months of life. Potential infections include otitis media and pneumonia caused by poor antibody response to capsular polysaccharides, and fungal and viral septicemias caused by T-cell dysfunction. A complete blood count could aid diagnosis; thrombocytopenia usually is in the 15,000 to 30,000/mm3 range, and platelets are typically small. In addition to eczema, these children have autoimmune disorders and a high incidence of lymphoma and other malignancies.

38.3 A 3-year-old boy presents with a swollen left knee and multiple bruises on the elbows, ankles, and shin. The parents insist that their son is very active and have noted that he "bruises easily" upon minor trauma. They appear to be very concerned by his condition. The child is found to have normal growth and development. The physical examination is consistent with the findings stated earlier. What is the most appropriate next step in evaluation of the patient? A Inform Child Protective Services. B Order a skeletal survey. C Obtain a detailed family history. D Order laboratory tests to rule out a bleeding disorder. E Consult a pediatric hematologist.

C. This presentation is typical for a bleeding diathesis likely hemophilia. In cases with a presentation concerning for physical abuse, it is imperative to get a detailed history including family history to rule out inherited bleeding disorders. The physical examination is consistent with bruises that are not particularly concerning for inflicted trauma. Evaluation with blood tests to rule out a bleeding disorder is indicated.

4.3 A 12-hour-old infant who has been feeding poorly becomes tachypneic with grunting. Which of the following initial tests has the lowest diagnostic yield? A Chest radiographs B Complete blood count C Urine culture D Blood culture E Glucose level

C. Urine cultures are not usually obtained in the workup of early onset sepsis. Urinary tract infections are rare in the first few days of life.

40.2 A mother notes her 6-week-old son's umbilical cord is still attached. His activity and intake are normal; he has had no illness or fever. Delivery was at term without problems. His examination is notable for a cord without evidence of separation and a shallow, 0.5-cm ulceration at the occiput without discharge or surrounding erythema. The mother declares that the "sore," caused by a scalp probe, has been slowly healing since birth and was deemed unremarkable at his 2-week checkup. Which of the following is consistent with this child's likely diagnosis? A Defective humoral response B Functional leukocyte adherence glycoproteins C Marked neutrophilia D Normal wound healing E Purulent abscess formation

C. You suspect leukocyte adhesion deficiency (LAD) as the etiology of this child's problem. LAD is an inheritable disorder of leukocyte chemotaxis and adherence characterized by recurring sinopulmonary, oropharyngeal, and cutaneous infections with delayed wound healing. Neutrophilia is common with white blood cell (WBC) counts typically more than 50,000 cells/mm3. Severe, life-threatening infection is possible with Staphylococcus species, Enterobacteriaceae, and Candida species. Good skin and oral hygiene are important; broad-spectrum antimicrobials and surgical debridement are early considerations with infection.

10.4 A 10-month-old child is seen in follow-up after a pediatric gastroenterologist visit for poor weight gain. Her current length is at the 10th percentile, head circumference is at the 50th percentile, and weight is less than the 5th percentile. The examination is unremarkable except for small size. She was sent to the gastroenterologist about a month prior by a colleague after her weight was noted to have dropped from the 50th percentile on her 6-month-old visit to less than the 5th percentile on her 9-month-old visit. Perinatal history is unremarkable. Feeding is via breast and bottle with a standard milk-based formula; the quantity of feeds reported seems sufficient and no excessive spit-up is reported. Various table foods are eaten and reportedly tolerated well. The child has had no recent or recurring illness. At the gastroenterology visit, a comprehensive array of laboratory and imaging studies were performed, as outlined in the referral letter, and were unhelpful in diagnosing the cause of the FTT. His recommendations at that time were to begin a 1-month's trial of a 24 cal/oz formula and follow-up with you for a weight check. Which of the following is the best next step in her care? A Commence caloric supplementation with 27 cal/oz formula. B Counsel family regarding diet and schedule follow-up weight check in 6 months. C Commence growth hormone administration. D Refer to nutritionist. E Admit to the hospital for FTT evaluation.

E. Without evident abnormality on examination and with a presumed thorough outpatient FTT workup, this infant probably has nonorganic FTT. Of note, the infant is already on a calorie-fortified formula, in addition to breast milk. Although it might be appropriate to solicit advice from a nutritionist, the best response would be to consider admission to the hospital for a multidisciplinary assessment of her FTT. A 2- to 3-day stay to watch intake ('calorie count') and daily weights, while selected services comment on the patient and her family (social services, nutrition, pediatric gastroenterology), might uncover a nonorganic cause for her FTT (parental neglect). Growth hormone administration and further counseling regarding diet without close follow-up are not standard of care.

21.4 The investigation of an unexpected infant death includes a history, a postmortem examination, and which of the following? A DNA studies B Maternal drug screen C Analysis of parental electrocardiograms D A death scene investigation E Stool studies

D. A death scene investigation is crucial to rule out trauma, both intentional and accidental.

12.2 Which of the following would be the most likely radiographic finding in a child who has been diagnosed with calcium-deficient rickets? A Widening of the ribs B Shortening of the radius and ulna C Vertebral fissures and fractures D Prominence of the costochondral junctions E Increased densities in the long bone shafts

D. Common bony abnormalities associated with calcium-deficient rickets include delayed closure of fontanelles, craniotabes, frontal bossing, dental hypoplasia, rachitic rosary, widening of the wrists and ankles, softening of the ribs (which may demonstrate fractures), bowing of the tibia or fibula, cupping or fraying at the distal ends of long bones, and deformities of the long bone shafts such as pseudofractures, pathologic fractures, fissures, or radiolucent lines.

31.1 The parents of a 3-year-old child are worried about the child's apparent clumsiness with frequent falls and a waddling gait. The child had normal development of motor skills during the first year of life and has normal language development. Which of the following is consistent with Duchenne muscular dystrophy? A Female gender B Hypertrophy of the quadriceps C 22-year-old sister with Becker muscular dystrophy D Gower sign E Positive antinuclear antibodies in the blood

D. Duchenne muscular dystrophy is an X-linked recessive disease and is clinically evident only in males. Affected boys may have calf hypertrophy that occurs as a compensation for proximal muscle weakness. They will generally develop a Gower sign.

6.2 A woman presents for her first prenatal visit at 9 weeks of gestation. She reports that she is generally healthy, except that she has an outbreak of genital herpes approximately once per year. To prevent transmission of the virus to her infant, her physician should do which of following? A Prescribe her daily acyclovir. B Order titers to determine if the infection is HSV-1 or HSV-2. C Perform weekly genital viral cultures starting at 36-week gestation. D Perform a cesarean delivery if herpetic lesions or prodromal symptoms are present when labor has begun. E No change in management is indicated; the risk of infant transmission is low even if she has an outbreak at delivery.

D. Even though the viral transmission risk in the setting of a recurrent HSV outbreak is low, cesarean section is indicated if lesions are present at the time of delivery because of the severity of neonatal HSV disease. Surveillance cultures are not recommended; negative results a few days prior to delivery do not preclude a later outbreak, and results of analysis of a more recently obtained specimen may not be available. Either type 1 or type 2 HSV can cause neonatal infection and disease. Daily antiviral therapy has not been shown to prevent neonatal infection.

15.4 A 15-month-old boy is brought to the emergency room by his mother for increasing bloody diarrhea over the past 2 days. She reports he was febrile 3 days ago with several episodes of emesis that had resolved by yesterday. In the past 24 hours he has had more than 10 episodes of diarrhea, with the most recent appearing to consist of only blood with no identifiable stool mixed with it. On physical examination, he is somnolent. Which of the following is NOT part of his immediate management? A A bolus of isotonic (normal) saline B Measurement of prothrombin time (PT) and activated partial thromboplastin time (APTT) C Ultrasound of the abdomen D A colonoscopy E A complete blood cell count (CBC)

D. Given the child's somnolence, immediate intravascular volume is needed. Isotonic saline should be given until volume is restored and then based on the hemoglobin and clotting studies, he may need red blood cells or plasma. His clinical presentation could represent intussusception and if seen on ultrasound, surgical consultation is needed. Colonoscopy would not be indicated until the patient is stabilized.

17.1 You are performing a routine 2-week well-child visit. The mother had routine prenatal care with no complications. During delivery, prolapse of the cord necessitated an emergent Cesarean section. The Apgar scores were 3, 7, and 9. The remainder of the hospital course was routine. The mother reports that her niece has cerebral palsy (CP). She asks about the risk for her baby having CP and you inform her that: A Having a family history of CP makes it likely that her child will also have it. B The low Apgar score is a risk for her child to develop CP. C Your examination today will determine if her infant has CP. D Most children with CP do not have a genetic risk, and at each well-child visit, you will continue to perform a careful developmental assessment. E You will refer her infant to a pediatric neurologist to do further workup.

D. Most children with CP do not have identifiable risk factors. Even in the presence of a risk factor, many children do not develop CP. The Apgar score at 1 minute reflects the neonatal environment immediately prior to birth; it does not predict an infant's neurologic outcome. CP cannot be ruled out on the basis of a normal neonatal physical examination. It is difficult to diagnose in children younger than 6 months. Assessing a child's development at each checkup is important for detecting abnormalities. If there is abnormal development or an abnormal examination finding, then referral to a neurologist may be appropriate.

34.4 At 3 weeks of life a former 33-week gestation infant develops poor feeding, abdominal distension, vomiting, and bloody stools. He has also had several apneic episodes and temperature instability overnight. Which of the following abnormalities would you expect to see on imaging? A "Double bubble" sign B Absence of colonic gas C Decreased or absent air in the rectum with dilated loops of bowel proximally D Air in the wall of the small intestine E Target sign

D. Necrotizing enterocolitis (NEC) can be mistaken for malrotation in young premature infants. Pneumatosis intestinalis (air in the wall of the small bowel) is a characteristic radiographing finding and aids in distinguishing NEC from malrotation and other abdominal pathology.

35.4 A 5 year old presents to clinic with abdominal pain and vomiting for the past 3 days. He has been afebrile but the mother reports strong smelling urine during that time. A urinalysis is obtained via catheterized sample and is significant for large leukocyte esterase and nitrates. The specimen is sent for culture and grows more than 100,000 CFU gram-negative rods. The mother denies any previous history of UTI. Which of the following is the next appropriate step in management for this patient? A Obtain a voiding cystourethrogram. B Obtain a cystoscopy. C Obtain a radionuclide scan using 99mTc dimercaptosuccinic acid (DMSA) scan to assess the extent of renal parenchymal damage. D Prescribe antimicrobial based on sensitivity.

D. Older children who do not have a previous history of UTIs can be treated on an outpatient basis with oral antibiotics. However, children aged 2 to 24 months, children with recurrent UTIs, febrile UTIs, or pyelonephritis should undergo evaluation for vesicoureteral reflux in addition to being treated with antimicrobials. These patients should undergo renal ultrasound to assess for renal anomalies. Patients with abnormalities on renal ultrasound should then have a VCUG to assess for vesicoureteral reflux where the presence of posterior urethral valves can be identified. If posterior urethral valves are detected on VCUG, a cystoscopy should be done to confirm the diagnosis and allow for ablation.

30.2 A 2-year-old boy who had a simple brief febrile seizure comes to your clinic a day after his ED visit. He is currently afebrile, is happily pulling the sphygmomanometer off the wall, and is taking antibiotics for an ear infection diagnosed the previous day. His mother wants to know what to expect in the future regarding his neurologic status. You correctly tell her which of the following? A He has no risk of further seizures because he was age 2 years at the time of his first febrile seizure. B He will need to take anticonvulsant medications for 6 to 12 months to prevent further seizure activity. C You want to schedule an EEG and a magnetic resonance scan of his head. D Although he does have a risk of future febrile convulsions, seizures of his type are generally benign and he is likely to outgrow them. E This is an isolated disorder, and his children will not have seizures.

D. Part of the anticipatory guidance for parents of children with febrile seizures is to impress upon them that the child may have another febrile seizure; it is similarly important to emphasize the usual benign nature of this condition. In a simple febrile seizure, imaging and EEG generally are not recommended, nor are prophylactic anticonvulsants. Because febrile seizures seem to have a genetic basis, it is possible that your patient's children will also have febrile seizures.

32.1 A mother brings her 2-week-old son to the clinic for a well-baby visit. Her only concern is a rash on his face and scalp that began a week earlier. Examination reveals a healthy white male child with normal vital signs and a normal examination except for yellowish, waxy-appearing, adherent plaques on the scalp, forehead, cheeks, and nasolabial folds. Which of the following therapies is appropriate for this condition? A High-potency steroid B Mupirocin C Acyclovir D Ketoconazole E Tacrolimus

D. Seborrheic dermatitis presents in infancy and adolescence. The chronic, symmetrical eruption, characterized by overproduction of sebum, affects the scalp, forehead, retroauricular region, auditory meatus, eyebrows, cheeks, and nasolabial folds. More commonly known as "cradle cap" in infants, this self-limited eruption typically develops between 2 and 3 months of age primarily on the scalp. The scale is yellow and waxy, and typically comes off with daily shampooing. The scale may be loosened with a small amount of oil. In infants who do not respond to shampooing with baby shampoo, an antidandruff shampoo containing antifungal medication (Nizoral) or seleniummay help, as will low-to-medium-potency topical corticosteroids.

15.2 A 2-year-old girl presents to the emergency room with her second episode of bloody stool. The mother has brought in the diaper which is filled with about one-fourth cup of brick-colored stool. The first episode of hematochezia occurred 6 months ago but the stools returned to normal within 2 days. The mother denies constipation, diarrhea, and fever. On physical examination, the child is awake and alert, her heart rate is 150 beats/min, she has no tenderness on palpation of the abdomen, and rectal examination is normal except her fecal occult blood test (FOBT) is positive. What would be the next best steps in management? A Prescribe a stool softener and have the child follow-up with her pediatrician the next day. B Reassure the mother that the cause of bleeding is a benign polyp and no treatment is needed. C Inquire more about the amount of ibuprofen the girl has been taking and prescribe omeprazole. D Administer a normal saline bolus and order a Meckel scan. E Ask the mother about any family history of Crohn disease or ulcerative colitis, and send an erythrocyte sedimentation rate (ESR).

D. The child is tachycardic and the amount of GI bleeding is significant. She requires her intravascular volume to be restored. Given her age, the most likely cause of her painless rectal bleeding is Meckel diverticulum.

33.1 A mother recently feels a mass in the abdomen of her 4-year-old son during a bath and brings him to your clinic for evaluation. He has no history of emesis, abnormal stooling, or abdominal pain. Physical examination reveals a resting blood pressure of 130/88 mm Hg, heart rate of 82 beats/min, pallor, and a firm left-sided abdominal mass that doesn't cross the midline. Which of the following is the most likely explanation for these findings? A Constipation B Intussusception C Neuroblastoma D Wilms tumor E Volvulus

D. The scenario presented is typical for Wilms tumor. Beyond abdominal imaging, checking a urinalysis for hematuria, metabolic panel for renal or hepatic dysfunction, and complete blood count for anemia should be considered in the workup of Wilms tumor.

4.2 A 25-day-old female infant is brought to the emergency department for fever of 101°F (38.3°C) at home. The baby was born vaginally at full term and was appropriate for gestational age. Maternal GBS was negative. Apgar scores were 8 and 9. The mother noticed the baby has had decreased feeding over the previous few days and has been sleeping more. Which of the following is the most appropriate initial choice of antibiotics for this infant? A Oral amoxicillin B Vancomycin C Ampicillin D Ampicillin and cefotaxime E Ampicillin and gentamicin

D. This patient may have late-onset bacterial infection, likely GBS; she should be admitted for sepsis evaluation and IV antibiotics. The best initial treatment in this age group is broad-spectrum antibiotics such as ampicillin and cefotaxime. If cultures are positive for GBS, antibiotic therapy can be narrowed to penicillin G.

13.3 Evaluation of a 9-year-old girl with known SCD reveals right-sided weakness and slurring of speech without any associated pain. Her vital signs are within normal limits for her age. Which of the following screening tests might have helped to prevent this complication of SCD? A Annual CT of head B Annual MRI of head C Biannual check of hemoglobin level D Annual transcranial Doppler ultrasound E Annual neurologic assessments during well-child checks

D. Transcranial Doppler ultrasound should initially be performed at 2 years of age. If normal, it should be repeated annually until the patient is 16 years old. If two ultrasounds are abnormal, transfusion therapy typically is initiated and continued indefinitely to help prevent stroke. CT and MRI without any signs of stroke are not indicated. A change in hemoglobin level does not indicate potential stroke in a patient with SCD, but may be concerning for infection or aplastic crisis. Although neurological examinations are an important part of any physical examination, changes in the examination would indicate an already evolving process, rather than help to predict the potential for future disease.

21.1 Which of the following features makes sudden infant death syndrome (SIDS) likely as the cause of a sudden death? A An infant found with a bulging fontanelle and facial bruise. B An 18-month-old girl who had a prior sibling that at 1 year of age also died suddenly and unexpectedly. C A 5-month-old infant with dysmorphic features and an enlarged heart found on postmortem examination. D A 3-month-old boy whose parents smoke in the home but were using a high-efficiency particulate arrestance (HEPA) air purifier in his room, a baby monitor, placing him on his side to sleep so he wouldn't aspirate any refluxed formula, and using a special foam wedge pillow to keep him in that position. E All the above features make SIDS likely as the cause of a sudden death.

D. Unfortunately, no specific product has been proven to reduce the risk of SIDS. The child's age, gender, and exposure to tobacco smoke are the main features that make SIDS a possible cause of death. Physical signs of any trauma make intentional injury and infanticide the most likely causes of death, whereas anatomic defects make cardiac or metabolic causes likely. The occurrence of a genetic susceptibility to SIDS within a family is exceedingly rare and SIDS cannot be the diagnosis if the patient is not an infant, that is, older than 12 months.

36.3 A 3-month-old boy presents to the emergency room with his father with a complaint of right arm swelling. The father states he fell asleep on the couch and when he awoke the patient was crying and lying next to the playpen in which he had been placed earlier. Which of the following is the most appropriate next step in the management of this child? A Splint the arm and discharge the patient. B Obtain radiographs of the right arm only. C Supinate the affected arm with the elbow in flexed position. D Order a skeletal survey. E Inform Child Protective Services.

D.A 3-month-old child would not be able to climb out of a playpen to sustain the injuries his father stated. A high likelihood for child abuse in this scenario exists. The next step is to perform a detailed history and physical examination, followed by a skeletal survey (full-body radiographs) to assess for old or new injuries in the infant. If further questions remain, admission to the hospital and notification to Child Protective Services are indicated.

31.2 Which of the following is the best screening test for the child discussed in Question 31.1? A Muscle biopsy B Measurement of serum creatinine C Electromyogram D Blood analysis for antinuclear antibodies E Measurement of serum creatine kinase level

E. A definitive diagnosis can be made by using muscle biopsy tissue, but serum creatine kinase measurement is preferred because it is less invasive and results can be obtained rapidly. Electromyography will reveal nonspecific myopathy.

16.3 A 5-year-old girl developed high fever, ear pain, and vomiting a week ago. She was diagnosed with OM and started on amoxicillin-clavulanate. On the third day of this medication she continued with findings of OM, fever, and pain. She received ceftriaxone intramuscularly and switched to oral cefuroxime. Now, 48 hours later, she has fever, pain, and no improvement in her OM; otherwise she is doing well. Which of the following is the most logical next step in her management? A Addition of intranasal topical steroids to the oral cefuroxime B Adenoidectomy C High-dose oral amoxicillin D Oral trimethoprim-sulfamethoxazole E Tympanocentesis and culture of middle ear fluid

E. After failing several antibiotic regimens, tympanocentesis and culture of the middle ear fluid are indicated.

44.3 You are seeing 10-year-old girl for evaluation of short stature. Which of the following are important to the workup? A History and physical including examination of growth chart B Parental height, growth pattern, and timing of puberty C Birth weight and length D Assessment for underlying systemic disease E All of the above

E. All of the above should be investigated. The differential diagnosis for growth delay is broad. A careful history and physical examination is the most important initial step. Any signs or symptoms of chronic disease would then warrant further evaluation. If a child's growth chart is unavailable, questions about a child's clothing or shoe size changes may provide valuable information. Obtaining history about parental growth pattern and their onset of puberty also provides important diagnostic clues to the child's etiology of short stature.

45.1 A 5-year-old girl has bilateral breast development that was first noticed 6 months ago. She takes no medications, and no source of exogenous estrogen is present in the home. Family history is unremarkable. Physical examination reveals a girl who is at the 50th percentile for height and weight, with normal blood pressure, normal skin without oiliness, Tanner stage II breasts, soft abdomen without palpable masses, no body odor, no pubic/axillary hair, and mild estrogenization of the vagina. Which of the following is the most likely explanation for the child's breast development? A Adrenal tumor B Central precocious puberty C Congenital adrenal hyperplasia D Premature adrenarche E Premature thelarche

E. All this child's findings are estrogen related and represent premature thelarche, a form of incomplete precocious puberty. She has no virilization. Postulated premature thelarche causes include ovarian cysts and transient gonadotropin secretion. No treatment is necessary. Bone age would be normal.

30.3 A 10-month-old boy presents to the ED with a 1-day history of fever to 104°F (40°C), increased irritability, decreased breast-feeding, and refusal of solid foods. The parents brought him in after two 30-second episodes of generalized jerking that occurred over a 20-minute span. Your examination reveals an awake but lethargic infant. The anterior fontanelle is flat, the tympanic membranes and oropharynx are moist and not erythematous, the lungs are clear, and the heart and abdominal examinations are normal. He has no focal neurologic findings. Which of the following is the best next step in management? A Intravenous ceftriaxone B Admission overnight for observation C Computed tomography of the head D Discharge from ED to follow up with his primary care provider in 24 hours E Lumbar puncture

E. Although this child ultimately may be diagnosed as having had a simple febrile seizure, the patient's age (<1 year) precludes a reliable neck examination. An LP is required to evaluate the child for meningitis. Administering antibiotics before the LP (or other cultures are obtained) is inadvisable unless the patient's condition is such that he would not tolerate the procedure.

1.3 A 37-week gestation boy is born after an uncomplicated pregnancy to a 33-year-old mother. At birth he was lethargic and had an HR of 40. Oxygen was administered via bag and mask, and he was intubated; his HR remained at 40 beats/min. Which of the following is the most appropriate next step? A Administer IV bicarbonate. B Administer IV atropine. C Administer IV epinephrine. D Administer IV calcium chloride. E Begin chest compressions.

E. Based on the neonatal resuscitation algorithm, if the HR is still less than 60 beats/min despite positive-pressure ventilation (PPV) with 100% oxygen, then chest compressions are given for 30 seconds. If the HR is still less than 60 beats/min, then drug therapy (usually epinephrine) is indicated.

1.1 A female infant is born through emergency cesarean section to a 34-year-old mother whose pregnancy was complicated by hypertension and abnormal fetal heart monitoring. At delivery she is covered in thick, green meconium and is limp, apneic, and bradycardic. Which of the following is the best first step in her resuscitation? A Administer IV bicarbonate. B Administer IV naloxone. C Initiate bag-and-mask ventilation. D Initiate chest compressions immediately. E Intubate with an endotracheal tube and suction meconium from the trachea.

E. If meconium is present during delivery, a decision whether to intubate and suction the trachea will depend on the newborn's degree of vigor (vigorous is defined as strong respiratory efforts, good muscle tone, and a heart rate greater than 100 beats/min). If the baby is nonvigorous as in this scenario, direct suctioning of the trachea is indicated before many breaths occur which will reduce the chance of forcing meconium into the lower airways and the development of meconium aspiration syndrome (obstructed airways by meconium, gas exchange impairment, and respiratory failure). When meconium is present and the baby is vigorous, a simple bulb syringe or large-bore suction catheter can be used to clear the airway and routine care can proceed.

3.5 A 12-day-old male infant presents to clinic for well-child check. His mother is concerned because she has noticed that "his eyes are turning yellow." She reports that he is a "good eater" and is proud to report that he is exclusively breast-fed. The patient has gained weight since birth and is voiding and stooling appropriately. What is the most likely cause of his jaundice? A Physiologic jaundice B Crigler-Najjar C Breast-feeding jaundice D Breast milk jaundice E TORCH infection

E. Patient is above birth weight and is exclusively breast-fed.

4.1 An infant was born at 36 weeks of gestation to a 30-year-old G3P2 mother via spontaneous vaginal delivery. Rupture of membranes occurred 15 hours prior to delivery. Birth weight is 4000 g, and Apgar scores were 6 and 9 at 1 and 5 minutes, respectively. Which of the following factors places this infant at greatest risk for sepsis? A Maternal age B Gestational size C Apgar score D Length of time membranes were ruptured E Gestational age

E. Prematurity places this baby at greater risk for sepsis. Young maternal age, low birth weight, rupture of membranes greater than 18 hours, initial Apgar less than 5, and maternal fever are additional risk factors for sepsis.

1.2 A term male infant is delivered vaginally to a 22-year-old mother. Immediately after birth he is noted to have a scaphoid abdomen, cyanosis, and respiratory distress. Heart sounds are heard on the right side of the chest, and the breath sounds seem to be diminished on the left side. Which of the following is the most appropriate next step in his resuscitation? A Administer IV bicarbonate. B Administer IV naloxone. C Initiate bag-and-mask intubation. D Initiate chest compressions immediately. E Intubate with an endotracheal tube.

E. The case describes diaphragmatic hernia. Because of herniated bowel in the chest, these children often have pulmonary hypoplasia. Bag-and-mask ventilation will cause accumulation of bowel gas (which is located in the chest) and further respiratory compromise. Therefore, endotracheal intubation is the best course of action.

40.4 You are called urgently to examine a term, 2-hour-old newborn with temperature instability, difficulty with feeding, and a suspected seizure. He has atypical facies (wide-set eyes, a prominent nose, and a small mandible), a cleft palate, and a holosystolic murmur. A chest radiograph reveals a boot-shaped heart. Which of the following is consistent with this infant's likely diagnosis? A Hypercalcemia B Chromosomal duplication C Parathyroid hyperplasia D Hypophosphatemia E Thymic aplasia

E. The child in the question has typical features of DiGeorge syndrome, caused by a 22q11 microdeletion. This syndromic immunodeficiency is characterized by decreased T-cell production and recurring infection. Findings include characteristic facies and velocardiofacial defects, such as ventricular septal defect and tetralogy of Fallot. Thymic or parathyroid dysgenesis can occur, accompanied by hypocalcemia and seizures. Developmental and speech delay are common in older patients.

37.4 A 3-year-old boy has pallor, lethargy, and decreased urine output. He was well until the preceding week, when he had fever, vomiting, and bloody diarrhea (now resolved). On examination, he is lethargic and has hepatosplenomegaly and scattered petechiae. Urinalysis reveals hematuria and proteinuria. Which of the following statements about his condition is accurate? A A complete blood (cell) count (CBC) is likely to reveal thrombocytosis. B Initial therapy includes systemic corticosteroids. C Empiric antimicrobial therapy for sepsis should be initiated. D An emergent oncology consultation for probable leukemia should be arranged. E Peripheral blood smear is likely to reveal helmet cells and burr cells.

E. This child has features of hemolytic-uremic syndrome (HUS), which frequently follows a bout of gastroenteritis; it has been associated with Escherichia coli 0157:H7, Shigella, and Salmonella. Patients have pallor, lethargy, and decreased urine output; some have hepatosplenomegaly, petechiae, and edema. Laboratory findings include hemolytic anemia and thrombocytopenia; peripheral blood smear demonstrates helmet cells, burr cells, and fragmented RBCs. Acute renal failure is manifested by hematuria, proteinuria, and an elevated serum creatinine level. Management is supportive with careful monitoring of renal and hematologic parameters; dialysis may be required.

22.2 A 3-month-old boy arrives in the emergency room in December with tachypnea and retractions. His mother reports he has nasal congestion, cough, and is not able to feed well. On auscultation, you note fine crackles and an end-expiratory wheeze with every intermittent wheeze. He is tachycardic to 190, and a 2/6 harsh holosystolic murmur is identified along the left sternal border. CXR shows cardiomegaly and increased pulmonary vascular markings. During venipuncture and IV insertion, he is crying vigorously and has an episode in which he appears slightly dusky. Which medication is indicated? A Albuterol B Racemic epinephrine C Indomethacin D Prostaglandin E Furosemide

E. This child is exhibiting congestive heart failure. Based on the accompanying murmur and CXR, the most likely etiology is a large VSD. Furosemide decreases the pulmonary congestion.

13.1 Appropriate advice for a mother of a 2-week-old child identified on newborn state screen to have sickle cell disease (SCD) includes which of the following? A Initiation of iron therapy B Emergent genetic testing of both parents for hemoglobinopathy status C Initiation of hydroxyurea therapy D Purchase of an apnea monitor E Enrollment in a comprehensive sickle cell program

E. This child must be enrolled in a comprehensive SCD program to ensure the best possible outcome. At 2 weeks of age, the child has no reason to be iron deficient, and combined with future blood transfusions that may be required, iron therapy could result in iron overload. The newborn state screen has shown the child to have SCD and that both parents have at least a single sickle cell gene; further testing of the family may be warranted, but not as an emergency. Hydroxyurea is used to increase the levels of fetal hemoglobin; this child in the first months of life already has significant quantities of that hemoglobin present. SCD is not an indication for an apnea monitor.

22.4 A previously healthy term infant suddenly develops respiratory distress on day 3 of life. An echocardiogram reveals coarctation of the aorta. Which of the following is the most appropriate treatment for immediate stabilization of this infant? A Digoxin B Furosemide C Albuterol D Racemic epinephrine E Prostaglandin therapy

E. This infant's symptoms started when his ductus arteriosus began to close. A continuous intravenous infusion of prostaglandin will keep the ductus open and allow blood flow to reach past the area of coarctation and perfuse the lower portion of the body. Surgery provides definitive repair.

33.3 A father presents his otherwise healthy 15-month-old daughter to the emergency center with cough, post-tussive emesis, and subjective fever over the past 3 days. He also thinks her abdomen has been hurting her. Diarrhea started yesterday, with "regular" stooling prior to this illness. She has been drinking well and recently had a wet diaper. Physical examination reveals normal vital signs, congested nares, shoddy neck lymphadenopathy, and a mildly distended and apparently tender abdomen without obvious guarding. Which of the following is the next best step in your evaluation? A Obtain abdominal computed tomography (CT). B Biopsy lymph node. C Collect 24-hour urine for catecholamines. D Admit to the hospital for exploratory laparotomy. E Reassure parent and await spontaneous resolution.

E. Upper respiratory tract infection symptoms, neck lymphadenopathy, and diarrhea are consistent with viremia; viral-mediated mesenteric lymph node enlargement can occur and cause nonspecific abdominal pain. Parental reassurance is adequate in this otherwise healthy child with classic viremia signs. An abdominal CT scan may show diffuse, mildly enlarged lymph nodes in mesenteric lymphadenitis, but imaging is rarely warranted unless an etiology for abdominal pain remains elusive.


संबंधित स्टडी सेट्स

Chapter 11: Additional Child Tax Credit & EITC

View Set

Math 7 Ch 5 Review: Probability and Solving Words Problems

View Set

Personal Auto Policy, Commercial Auto Policies, Workers Compensation, HO policies, Other Coverages needed to rework:, Bonds:, NFIP, Property and Casualty Exam, Property & Casualty - Insurance Regulations, Insurance License - Liberty Mutual

View Set

Fundamentals PrepU Chapter 25: Health Assessment

View Set